Maryville 611 patho exam 4

Ace your homework & exams now with Quizwiz!

Which statement is false concerning how abdominal pain is produced? a. Chemical mediators, such as histamine, bradykinin, and serotonin, produce abdominal pain. b. Edema and vascular congestion produce abdominal pain by stretching. c. Ischemia, caused by distention of bowel obstruction or mesenteric vessel thrombosis, produces abdominal pain. d. Low concentrations of anaerobes, such as Streptococci, Lactobacilli, Staphylococci, Enterobacteria, and Bacteroides, produce abdominal pain.

ANS: D Low concentrations of anaerobes are not typically a cause of abdominal pain.

Which anomaly is often associated with Wilms tumor? a. Renal anaplasia c. Anemia b. Aniridia d. Hypothyroidism

Aniridia (lack of an iris in the eye) is an anomaly often associated with Wilms tumor. Approximately 10% of children who have Wilms tumor also have a loss of other important genes and therefore have a number of congenital anomalies. The other anomalies listed are not associated with a Wilms tumor.

Congenital aganglionic megacolon (Hirschsprung disease) involves inadequate motility of the colon caused by neural malformation of which nervous system? a. Central c. Sympathetic b. Parasympathetic d. Somatic

B A malformation related only to the parasympathetic nervous system causes congenital aganglionic megacolon.

Which condition is capable of producing alveolar dead space? a. Pulmonary edema c. Atelectasis b. Pulmonary emboli d. Pneumonia

B A pulmonary embolus that impairs blood flow to a segment of the lung results in an area where alveoli are ventilated but not perfused, which causes alveolar dead space. Alveolar dead space is not the result of any of the remaining options.

Acute glomerulonephritis (AGN) may be accompanied by a positive throat or skin culture for which bacteria? a. Staphylococcus aureus c. Pseudomonas aeruginosa b. Streptococcus d. Haemophilus

B AGN may be accompanied by a positive throat or skin culture for Streptococcus. AGN is not associated with any of the other options.

Which statement best describes acute respiratory distress syndrome (ARDS)? a. An obstructive airway disease characterized by reversible airflow obstruction, bronchial hyperreactivity, and inflammation b. A pulmonary disease characterized by severe hypoxemia, decreased pulmonary compliance, and the presence of bilateral infiltrates on chest x-ray imaging c. A respiratory disorder involving an abnormal expression of a protein producing viscous mucus that lines the airways, pancreas, sweat ducts, and vas deferens d. A pulmonary disorder characterized by atelectasis and increased pulmonary resistance as a result of a surfactant deficiency

B ARDS is a condition that can result from either a direct or indirect pulmonary insult. It is defined as respiratory failure of acute onset characterized by severe hypoxemia that is refractory to treatment with supplemental oxygen, bilateral infiltrates on chest x-ray imaging, and no evidence of heart failure, as well as decreased pulmonary compliance. This selection is the only option that accurately describes ARDS.

Which statement about the advances in the treatment of respiratory distress syndrome (RDS) of the newborn is incorrect? a. Administering glucocorticoids to women in preterm labor accelerates the maturation of the fetus's lungs. b. Administering oxygen to mothers during preterm labor increases their arterial oxygen before the birth of the fetus. c. Treatment includes the instillation of exogenous surfactant down an endotracheal tube of infants weighing less than 1000 g. d. Using continuous positive airway pressure (CPAP) supports the infant's respiratory function

B Administering oxygen to the mother is not a valid treatment of RDS. The other statements provide correct information regarding RDS.

What is the most abundant class of plasma protein? a. Globulin c. Clotting factors b. Albumin d. Complement proteins

B Albumin (approximately 60% of total plasma protein at a concentration of about 4 g/dl) is the most abundant plasma protein.

What causes vesicoureteral reflux to occur in children? a. Children do not ask for help in urinating in a timely manner, and urine is forced up into the ureters. b. The submucosal segment of a child's ureter is short, making the antireflux mechanism inefficient. c. The trigone lying between the opening to the ureters and the urethra is underdeveloped in children. d. As the bladder fills in infants and children, it pulls the smooth lining of the transitional epithelium away from the ureters, making the reflux valves ineffective.

B Although reflux is considered abnormal at any age, the shortness of the submucosal segment of the ureter during infancy and childhood renders the antireflux mechanism relatively inefficient and delicate. The other options are not considered reasons for this reflux.

How does progressive nephrons injury affect angiotensin II activity? a. Angiotensin II activity is decreased. b. It is elevated. c. Angiotensin II activity is totally suppressed. d. It is not affected.

B Angiotensin II activity is elevated with progressive nephron injury. This selection is the only accurate identification of the effect of progressive nephron injury on angiotensin II activity.

Which T-lymphocyte phenotype is the key determinant of childhood asthma? a. Cluster of differentiation (CD) 4 T-helper Th1 lymphocytes b. CD4 T-helper Th2 lymphocytes c. CD8 cytotoxic T lymphocytes d. Memory T lymphocytes

B Asthma develops because the Th2 response (in which CD4 T-helper cells produce specific cytokines, such as interleukin [IL]-4, IL-5, and IL-13) promotes an atopic and allergic response in the airways. This selection is the only option that accurately identifies the appropriate T-lymphocyte phenotype.

Which immunoglobulin (Ig) may contribute to the pathophysiologic characteristics of asthma? a. IgA c. IgG b. IgE d. IgM

B Asthma is a familial disorder, and more than 100 genes have been identified that may play a role in the susceptibility of and the pathogenetic mechanisms that cause asthma, including those that influence the production of interleukin (IL)-4, IL-5, and IL-13; IgE; eosinophils; mast cells; adrenergic receptors; and leukotrienes. The pathophysiologic characteristics of asthma are not associated with the other immunoglobulins.

Which option shows the correct sequence of events after atelectasis develops in respiratory distress syndrome of the newborn? a. Increased pulmonary vascular resistance, atelectasis, hypoperfusion b. Hypoxic vasoconstriction, right-to-left shunt hypoperfusion c. Respiratory acidosis, hypoxemia, hypercapnia d. Right-to-left shunt, hypoxic vasoconstriction, hypoperfusion

B Atelectasis results in a decrease in tidal volume, causing alveolar hypoventilation and hypercapnia. Hypoxia and hypercapnia cause pulmonary vasoconstriction, which increases intrapulmonary resistance and shunting. This results in hypoperfusion of the lung and a decrease in effective pulmonary blood flow. This selection is the only option that identifies the correct sequence of events.

What term is used to identify an inflammation of the glans penis? a. Glanitis c. Priapism b. Balanitis d. Hydrocelitis

B Balanitis is the only term used to identify an inflammation of the glans penis

An infant suddenly develops abdominal pain, becomes irritable (colicky), and draws up the knees. Vomiting occurs soon afterward. The mother reports that the infant passed a normal stool, followed by one that looked like currant jelly. Based on these data, which disorder does the nurse suspect? a. Congenital aganglionic megacolon c. Malrotation b. Intussusception d. Volvulus

B Based on these data, the nurse should suspect intussusception. A single normal stool may be passed, evacuating the colon distal to the apex of the intussusception. After passing a normal stool, 60% of infants will pass "currant jelly" stools, which appear dark and gelatinous because of their blood and mucus content. Intussusception is the only option that describes the symptoms listed.

Considering the innervation of the circular muscles of the bladder neck, which classification of drug is used to treat bladder neck obstruction? a. β-Adrenergic blocking medications c. Parasympathomimetic medications b. α-Adrenergic blocking medications d. Anticholinesterase medications

B Because the bladder neck consists of circular smooth muscle with adrenergic innervation, detrusor sphincter dyssynergia may be managed by α-adrenergic blocking (antimuscarinic) medications. This selection is the only option capable of this specific function.

What changes to the hematologic system is related to age? a. Platelet adhesiveness decreases. b. Lymphocyte function decreases. c. Cellular immunity increases. d. Erythrocyte reproduction accelerates.

B Blood composition changes little with age. A delay in erythrocyte replenishment may occur after bleeding, presumably because of iron deficiency. Lymphocyte function appears to decrease with age. Particularly affected is a decrease in cellular immunity. Platelet adhesiveness probably increases with age.

Which statement is false concerning the skeletal alterations caused by chronic renal failure when the glomerular filtration rate (GFR) declines to 25% of normal? a. Parathyroid hormone is no longer effective in maintaining serum phosphate levels. b. The parathyroid gland is no longer able to secrete sufficient parathyroid hormone. c. The synthesis of 1,25-vitamin D3, which reduces intestinal absorption of calcium, is impaired. d. The synthesis of 1,25-vitamin D3, which impairs the effectiveness of calcium and phosphate resorption from bone by parathyroid hormone, is impaired.

B Bone and skeletal changes develop with alterations in calcium and phosphate metabolism (see Table 38-16). These changes begin when the GFR decreases to 25% or less. The combined effect of hyperparathyroidism and vitamin D deficiency can result in renal osteodystrophies (e.g., osteomalacia, osteitis fibrosa with increased risk for fractures). Other consequences of secondary hyperparathyroidism include soft-tissue and vascular calcification, cardiovascular disease, and, less commonly, calcific uremic arteriolopathy. The other options are true.

What is the reason breast cancer in men has such a poor prognosis? a. Breast cancer is extremely aggressive in men. b. Treatment is usually delayed as a result of late detection. c. Chemotherapies are not as effective in men. d. Breast tumors tend to be small and hard to isolate.

B Breast cancer is relatively uncommon in men, but it has a poor prognosis because men tend to delay seeking treatment until the disease is advanced. This selection is the only option that accurately identifies the reason breast cancer in men has a poor prognosis.

The progression of chronic bronchitis is best halted by which intervention? a. Regular use of bronchodilators b. Smoking cessation c. Postural chest drainage techniques d. Identification of early signs of infection

B By the time an individual seeks medical care for symptoms, considerable airway damage is present. If the individual stops smoking, then disease progression can be halted. If smoking is stopped before symptoms occur, then the risk of chronic bronchitis decreases considerably and eventually reaches that of nonsmokers. The other interventions, although appropriate, are not directed toward halting the progression of the disease process.

What term is used to describe the selective bulbous enlargement of the distal segment of a digit that is commonly associated with diseases that interfere with oxygenation of the blood? a. Edema c. Angling b. Clubbing d. Osteoarthropathy

B Clubbing is the selective bulbous enlargement of the end (distal segment) of a digit (finger or toe) (see Figure 35-1) and is commonly associated with diseases that interfere with oxygenation, such as bronchiectasis, cystic fibrosis, pulmonary fibrosis, lung abscess, and congenital heart disease. None of the remaining options are terms used to identify the condition described.

With a total hemoglobin of 9 g/dl, how many grams per deciliter of hemoglobin must become desaturated for cyanosis to occur? a. 3 c. 7 b. 5 d. 9

B Cyanosis generally develops when 5 g/dl of hemoglobin is desaturated, regardless of hemoglobin concentration.

What is the cause of reflux esophagitis? a. Immune response to gastroesophageal reflux b. Delayed gastric emptying c. Congenital anomaly d. Secretory response to gastroesophageal reflux

B Delayed gastric emptying contributes to reflux esophagitis by (1) lengthening the period during which reflux is possible and (2) increasing the acid content of chyme. None of the other options are accurate descriptions of the cause of reflux esophagitis.

The absence of parietal cells would prevent the absorption of an essential nutrient necessary to prevent which type of anemia? a. Iron deficiency c. Folic acid deficiency anemia b. Pernicious anemia d. Aplastic anemia

B Dietary vitamin B12 is a large molecule that requires a protein secreted by parietal cells into the stomach (intrinsic factor [IF]) to transport across the ileum. Defects in IF production lead to decreased B12 absorption and pernicious anemia. The other options are not the result of this process.

Considering host defense mechanisms, which element in the urine is bacteriostatic? a. High pH (alkaline urine) c. High glucose b. High urea d. High calcium

B Dilute urine washes out bacteria, and urine with higher urea concentrations (high osmolarity) is more bacteriostatic.

Which statement is false regarding the contributing factors of duodenal ulcers? a. Bleeding from duodenal ulcers causes hematemesis or melena. b. Gastric emptying is slowed, causing greater exposure of the mucosa to acid. c. The characteristic pain begins 30 minutes to 2 hours after eating when the stomach is empty. d. Duodenal ulcers occur with greater frequency than other types of peptic ulcers.

B Duodenal ulcers can be associated with altered mucosal defenses, rapid gastric emptying, elevated serum gastrin levels, or acid production stimulated by smoking. The other options provide correct information regarding duodenal ulcers.

Which condition involves an abnormally enlarged gas-exchange system and the destruction of the lung's alveolar walls? a. Transudative effusion c. Exudative effusion b. Emphysema d. Abscess

B Emphysema is abnormal permanent enlargement of gas-exchange airways (acini) accompanied by the destruction of alveolar walls without obvious fibrosis. The described mechanism is not associated with the other options.

Examination of the throat in a child demonstrating signs and symptoms of acute epiglottitis may contribute to which life-threatening complication? a. Retropharyngeal abscess c. Rupturing of the tonsils b. Laryngospasms d. Gagging induced aspiration

B Examination of the throat may trigger laryngospasm and cause respiratory collapse. Death may occur in a few hours. This selection is the only option that accurately identifies the life-threatening complication that can result from an examination of the throat of a child who demonstrates the signs and symptoms of acute epiglottitis.

In immunoglobulin G (IgG) nephropathies such as glomerulonephritis, IgG is deposited in which location? a. Juxtamedullary nephrons b. Glomerulus basement membranes c. Mesangium of the glomerular capillaries d. Parietal epithelium

B Glomerulonephritis develops with the deposition of antigen-antibody complexes (IgG, immunoglobulin A [IgA], and C3 complement) in the glomerulus, or the antigen may be trapped within the glomerulus and immune complexes formed in situ. Immunofluorescence microscopy shows lumpy deposits of IgG and C3 complement on the glomerular basement membrane (see Figure 39-5). When considering IgG nephropathies, the only location of the IgG immunoglobulins is the correct option.

What term is used to identify frank bleeding of the rectum? a. Melena c. Occult bleeding b. Hematochezia d. Hematemesis

B Hematochezia is the only available option that is associated with frank bright red or burgundy blood from the rectum.

Which statement is true regarding hypoxemia? a. Hypoxemia results in the increased oxygenation of arterial blood. b. Respiratory alterations cause hypoxemia. c. Hypoxemia results in the decreased oxygenation of tissue cells. d. Various system changes cause hypoxemia.

B Hypoxemia, or reduced oxygenation of arterial blood (PaO2), is caused by respiratory alterations, whereas hypoxia, or reduced oxygenation of cells in tissues, may be caused by alterations of other systems as well.

An intestinal obstruction at the pylorus or high in the small intestine causes metabolic alkalosis by causing which outcome? a. Gain of bicarbonate from pancreatic secretions that cannot be absorbed b. Excessive loss of hydrogen ions normally absorbed from gastric juices c. Excessive loss of potassium, promoting atony of the intestinal wall d. Loss of bile acid secretions that cannot be absorbed

B If the obstruction is at the pylorus or high in the small intestine, then metabolic alkalosis initially develops as a result of excessive loss of hydrogen ions that normally would be reabsorbed from the gastric juices. This selection is the only option that accurately describes the cause of metabolic alkalosis in this situation.

Which type of croup is most common? a. Bacterial c. Fungal b. Viral d. Autoimmune

B In 85% of children with croup, a virus is the cause, most commonly parainfluenza. However, other viruses such as influenza A or respiratory syncytial virus (RSV) also can cause croup

What is the pathophysiologic process responsible for the autoimmune disorder of hemolytic-uremic syndrome (HUS)? a. Immunoglobulin A (IgA) coats erythrocytes that are destroyed by the spleen, and remnants are excreted through the kidneys. b. Verotoxin from Escherichia coli is absorbed from the intestines and damages erythrocytes and endothelial cells. c. Endotoxins from E. coli block the erythropoietin produced by the kidneys, which reduces the number of erythrocytes produced by the bone marrow. d. Failure of the nephron to filter urea increases the blood urea nitrogen, which binds to erythrocytes that are subsequently destroyed by the spleen.

B In HUS, verotoxin from E. coli is absorbed from the intestines, the glomerular arterioles become swollen, and these narrowed vessels damage erythrocytes as they pass through. HUS is responsible for causing a cascade of effects, including lysis of glomerular capillary endothelial cells. The other options are not relevant.

What is the most common predisposing factor to obstructive sleep apnea in children? a. Chronic respiratory infections c. Obligatory mouth breathing b. Adenotonsillar hypertrophy d. Paradoxic breathing

B In otherwise healthy children, the most common predisposing factor is adenotonsillar hypertrophy, which causes physical impingement on the nasopharyngeal airway. The other options are not associated with obstructive sleep apnea in children.

Which pleural abnormality involves a site of pleural rupture that acts as a one-way valve, permitting air to enter on inspiration but preventing its escape by closing during expiration? a. Spontaneous pneumothorax c. Open pneumothorax b. Tension pneumothorax d. Secondary pneumothorax

B In tension pneumothorax, the site of pleural rupture acts as a one-way valve, permitting air to enter on inspiration but preventing its escape by closing up during expiration. As more and more air enters the pleural space, air pressure in the pneumothorax begins to exceed barometric pressure. None of the other options result from the pathologic condition described.

What is the role of collagen in the clotting process? a. Initiates the clotting cascade. c. Stimulates fibrin. b. Activates platelets. d. Deactivates fibrinogen.

B In the clotting process, collagen provides a particularly strong stimulus to activate platelets. Collagen does not bring about any of the other options.

Pulmonary edema in acute respiratory distress syndrome (ARDS) is the result of an increase in: a. Levels of serum sodium and water c. Capillary hydrostatic pressure b. Capillary permeability d. Oncotic pressure

B Increased capillary permeability, a hallmark of ARDS, allows fluids, proteins, and blood cells to leak from the capillary bed into the pulmonary interstitium and alveoli. The resulting pulmonary edema and hemorrhage severely reduce lung compliance and impair alveolar ventilation. The other options would not trigger ARDS-associated pulmonary edema.

Clinical manifestations that include unexplained weight loss, dyspnea on exertion, use of accessory muscles, and tachypnea with prolonged expiration are indicative of which respiratory disorder? a. Chronic bronchitis c. Pneumonia b. Emphysema d. Asthma

B Individuals with emphysema usually have dyspnea on exertion that later progresses to significant dyspnea, even at rest (see Table 35-3). Little coughing and very little sputum are produced. The individual is often thin, has tachypnea with prolonged expiration, and must use accessory muscles for ventilation. The anteroposterior diameter of the chest is increased (barrel chest), and the chest has a hyperresonant sound with percussion. The described symptoms are not associated with any of the other options.

Why is prolonged diarrhea more severe in children than it is in adults? a. Less water is absorbed from the colon in children. b. Fluid reserves are smaller in children. c. Children have a higher fluid volume intake. d. Children have diarrhea more often than adults.

B Infants have low fluid reserves and relatively rapid peristalsis and metabolism. Therefore the danger of dehydration is great. This selection is the only option that correctly identifies the reason prolonged diarrhea is more severe in children.

infertility is defined as the inability to conceive after how many months of unprotected intercourse with the same partner? a. 6 c. 18 b. 12 d. 24

B Infertility is defined as the inability to conceive after 1 year of unprotected intercourse with the same partner.

Which nutrients are necessary for hemoglobin synthesis? a. Protein and niacin c. Cobalamin (vitamin B12) and folate b. Iron and vitamin B6 (pyridoxine) d. Pantothenic acid and vitamin C

B Iron and B6 (pyridoxine) are necessary for hemoglobin synthesis (see Table 27-6). The remaining options are not necessary for hemoglobin synthesis.

What theory is used to describe the cause of endometriosis? a. Obstruction within the fallopian tubes prevents the endometrial tissue from adhering to the lining of the uterus. b. Endometrial tissue passes through the fallopian tubes and into the peritoneal cavity and remains responsive to hormones. c. Inflammation of the endometrial tissue develops after recurrent sexually transmitted diseases. d. Endometrial tissue lies dormant in the uterus until the ovaries produce sufficient hormone to stimulate its growth.

B It has been proposed that endometriosis is caused by the implantation of endometrial cells during retrograde menstruation, during which menstrual fluids move through the fallopian tubes and empty into the pelvic cavity (see Figure 24-16). Similar to normal endometrial tissue, the ectopic (out of place) endometrium responds to the hormonal fluctuations of the menstrual cycle. Of the available options, this answer is the only accepted theory for the cause of endometriosis.

Which form of iron (Fe) can be used in the formation of normal hemoglobin? a. Fe+ c. Fe3+ b. Fe2+ d. Fe4+

B It is crucial that the iron be correctly charged; only reduced ferrous iron (Fe2+) can bind oxygen in the lungs and release it in the tissues.

What type of diarrhea is a result of lactase deficiency? a. Motility c. Secretory b. Osmotic d. Small-volume

B Malabsorption related to lactase deficiency, pancreatic enzyme or bile salt deficiency, small intestine bacterial overgrowth, and celiac disease cause osmotic diarrhea. None of the other options are associated with lactase deficiencies.

Which term is used to identify an intestinal obstruction caused by meconium formed in utero that is abnormally sticky and adheres firmly to the mucosa of the small intestine? a. Meconium cecum c. Meconium obstruction b. Meconium ileus d. Meconium vivax

B Meconium ileus is the only term used to identify an intestinal obstruction caused by meconium formed in utero that is abnormally sticky and adheres firmly to the mucosa of the small intestine, resisting passage beyond the terminal ileum. The cause is usually a lack of digestive enzymes during fetal life.

Which laboratory test is considered adequate for an accurate and reliable diagnosis of gonococcal urethritis in a symptomatic man? a. Ligase chain reaction (LCR) c. Polymerase chain reaction (PCR) b. Gram-stain technique d. DNA testing

B Microscopic evaluation of Gram-stained slides of clinical specimens is deemed positive for Neisseria gonorrhoeae if gram-negative diplococci with the typical "kidney bean" morphologic appearance are found inside polymorphonuclear leukocytes. Such a finding is considered adequate for the diagnosis of gonococcal urethritis in a symptomatic man. The other options are not relevant to the diagnosis of this condition.

How many stools per day are considered the upper limits of normal? a. Two c. Five b. Three d. Seven

B More than three stools per day is considered abnormal.

The mutation of which gene is an early event associated with the pathogenetic origin of esophageal cancer? a. K-ras mutation c. myc b. TP53 d. HER2

B Mutation of the TP53 gene is an early event associated with esophageal cancer. This selection is the only mutation from among the provided options.

What is the cause of gastroesophageal reflux disease? a. Excessive production of hydrochloric acid b. Zone of low pressure of the lower esophageal sphincter c. Presence of Helicobacter pylori in the esophagus d. Reverse muscular peristalsis of the esophagus

B Normally, the resting tone of the lower esophageal sphincter maintains a zone of high pressure that prevents gastroesophageal reflux. In individuals who develop reflux esophagitis, this pressure tends to be lower than normal from either transient relaxation or a weakness of the sphincter. This selection is the only option that accurately describes the cause of gastroesophageal reflux disease.

Which type of pulmonary disease requires more force to expire a volume of air? a. Restrictive c. Acute b. Obstructive d. Communicable

B Obstructive pulmonary disease is characterized by airway obstruction that is worse with expiration. Either more force (i.e., the use of accessory muscles of expiration) or more time is required to expire a given volume of air. The other options are not associated with a need for an increase of force to expire air

Considering the pathophysiologic characteristics of primary amenorrhea, what anatomic structure is involved in compartment IV? a. Vagina c. Ovary b. Hypothalamus d. Anterior pituitary

B Of the options available, only compartment IV disorders include central nervous system (CNS) conditions, in particular hypothalamic disorders.

Sitting up in a forward-leaning position generally relieves which breathing disorder? a. Hyperpnea c. Apnea b. Orthopnea d. Dyspnea on exertion

B Of the options available, only orthopnea is generally relieved by sitting up in a forward-leaning posture or supporting the upper body on several pillows.

Which substance is used to correct the chronic anemia associated with chronic renal failure? a. Iron c. Cobalamin (vitamin B12) b. Erythropoietin d. Folate

B One of the most significant advances in the study of hematopoietic growth factors has been the development of erythropoietin for individuals with chronic renal failure. The other options are not associated with the treatment of chronic anemia.

During reactivation (release from latency), herpes virus genomes are transported through which nerves to the dermal surface? a. Somatic c. Autonomic b. Peripheral sensory d. Peripheral motor

B Only during reactivation are the viral genomes transported through the peripheral sensory nerves back to the dermal surface.

Which hepatitis virus is known to be sexually transmitted? a. A c. C b. B d. D

B Only hepatitis B virus (HBV) is known to be sexually transmitted.

Which medication compensates for the deficiency that occurs as a result of cystic fibrosis? a. Salt tablets c. Antihypertensives b. Pancreatic enzymes d. Antibiotics

B Pancreatic replacement enzymes are administered before or with meals, and high-calorie, high-protein diets with frequent snacks and vitamin supplements are used to treat the deficiency. These statements are not true of the other options.

What unique factor causes adolescent girls to have a high risk for sexually transmitted infections (STIs)? a. They are in an experimental phase with sexual intercourse and believe they are resistant to developing STIs. b. The adolescent cervix is immature and lacks immunity. c. The length of the vaginal canal is short in adolescents, allowing a greater concentration of microorganisms within the internal genitalia. d. In adolescent girls, the anus to the vaginal introitus are in close proximity.

B Partly, perhaps, because of risk-taking behavior (unprotected intercourse or selection of high-risk partners), many adolescents have an increased risk for STI exposure and infection. The unique factor for adolescent women is that they have a physiologically increased susceptibility to infection because of increased cervical immaturity and lack of immunity. The remaining options are not considered legitimate risk factors for STIs.

Which of the following are formed elements of the blood that are not cells but are disk-shaped cytoplasmic fragments essential for blood clotting? a. Monocytes c. Macrophages b. Platelets d. Erythrocytes

B Platelets (thrombocytes) are not true cells but are disk-shaped cytoplasmic fragments that are essential for blood coagulation and control of bleeding. This description is not accurate for any of the other options.

What causes pneumoconiosis? a. Pneumococci bacteria c. Exposure to asbestos b. Inhalation of inorganic dust particles d. Inhalation of cigarette smoke

B Pneumoconiosis represents any change in the lung caused by the inhalation of inorganic dust particles, which usually occurs in the workplace. Pneumoconiosis is not a result of any of the other options.

What medical term is used to identify the accumulation of air in the pleural space? a. Flail chest c. Pleural effusion b. Pneumothorax d. Exudate effusion

B Pneumothorax is the presence of air or gas in the pleural space caused by a rupture in the visceral pleura (which surrounds the lungs) or the parietal pleura and chest wall. The condition is not identified by any of the other options.

A person who has cholera would be expected to have which type of diarrhea? a. Osmotic c. Small volume b. Secretory d. Motility

B Primary causes of secretory diarrhea are bacterial enterotoxins, particularly those released by cholera or strains of Escherichia coli, and neoplasms, such as gastrinoma or thyroid carcinoma. None of the other options are associated with secretory diarrhea

The release of which chemical mediator causes primary dysmenorrhea? a. Leukotrienes c. Bradykinin b. Prostaglandins d. C-reactive protein

B Primary dysmenorrhea is painful menstruation associated with the release of prostaglandins in ovulatory cycles. This option is the only answer that accurately identifies the chemical mediator associated with dysmenorrhea.

Pulmonary edema usually begins to develop at a pulmonary capillary wedge pressure or left atrial pressure of how many millimeters of mercury (mm Hg)? a. 10 c. 30 b. 20 d. 40

B Pulmonary edema usually begins to develop at a pulmonary capillary wedge pressure or left atrial pressure of 20 mm Hg.

Which statement is false about the factors that facilitate the ascent of gonococci into the uterus and fallopian tubes? a. Ascent of gonococci is facilitated because the cervical plug disintegrates during menstruation. b. Ascent of gonococci is facilitated because the vaginal pH decreases to 2 or 3. c. Ascent of gonococci is facilitated because the uterine contractions may cause retrograde menstruation into the fallopian tubes. d. Ascent of gonococci is facilitated because the bacteria may adhere to sperm and be transported to the fallopian tubes.

B Several factors can facilitate the ascent of gonococci into the uterus and the fallopian tubes, where they cause pelvic inflammatory disease (PID). Among these factors are (1) disintegration of the cervical mucous plug and (2) a rise in vaginal pH greater than 4.5 during menstruation. The other options are accurate statements.

A woman diagnosed with trichomoniasis asks if her sexual partner should be treated as well. What is the appropriate response? a. Sexual partners should be treated only if symptoms are present. b. Sexual partners should be treated even if they are asymptomatic. c. Infections in men are self-limiting; therefore a male sexual partner does not require treatment. d. Sexual partners should be treated to prevent infection.

B Sexual partners, even if asymptomatic, are also treated and examined for coexisting sexually transmitted infections.

Which organism is responsible for the development of syphilis? a. Neisseria syphilis c. Haemophilus ducreyi b. Treponema pallidum d. Chlamydia trachomatis

B T. pallidum is the only cause of syphilis.

Which structure regulates eating behavior and energy metabolism? a. Anterior pituitary c. Posterior pituitary b. Hypothalamus d. Parietal lobe

B The arcuate nucleus (ARC) in the hypothalamus has two sets of neurons with opposing effects that interact to regulate and balance food intake and energy metabolism. This selection is the only option that regulates eating behavior and energy metabolism.

Between which months of age does sudden infant death syndrome (SIDS) most often occur? a. 0 and 1 c. 5 and 6 b. 2 and 4 d. 6 and 7

B The incidence of SIDS is low during the first month of life but sharply increases in the second month of life, peaking at 2 to 4 months and is unusual after 6 months of age.

Which statement about the late asthmatic response is true? a. Norepinephrine causes bronchial smooth muscle contraction and mucus secretion. b. The release of toxic neuropeptides contributes to increased bronchial hyperresponsiveness. c. The release of epinephrine causes bronchial smooth muscle contraction and increases capillary permeability. d. Immunoglobulin G initiates the complement cascade and causes smooth muscle contraction and increased capillary permeability.

B The late asthmatic response begins 4 to 8 hours after the early response when the release of toxic neuropeptides contributes to increased bronchial hyperresponsiveness. This selection is the only option associated with the late asthmatic response.

Where are Kupffer cells located? a. Kidneys c. Pancreas b. Liver d. Spleen

B The liver macrophages are the only location for Kupffer cells.

What is the most common cause of pulmonary edema? a. Right-sided heart failure c. Mitral valve prolapse b. Left-sided heart failure d. Aortic stenosis

B The most common cause of pulmonary edema is heart disease. When the left ventricle fails, filling pressures on the left side of the heart increase and cause a concomitant increase in pulmonary capillary hydrostatic pressure. The remaining options are not common triggers for pulmonary edema.

The risk of which cancer is greater if the man has a history of cryptorchidism? a. Penile c. Prostate b. Testicular d. Epididymal

B The risk of testicular cancer is 35 to 50 times greater in men with cryptorchidism or in those with a history of cryptorchidism than it is for the general male population. This is not true of the other options.

Fetal hematopoiesis occurs in which structure? a. Gut c. Bone marrow b. Spleen d. Thymus

B The spleen is the largest of the secondary lymphoid organs and the site of fetal hematopoiesis.

Which glomerular lesion is characterized by thickening of the glomerular capillary wall with immune deposition of immunoglobulin G (IgG) and C3? a. Proliferative c. Mesangial b. Membranous d. Crescentic

B The thickening of the glomerular capillary wall characterizes only membranous lesions.

Where in the brain is the vomiting center located? a. Hypothalamus c. Pons b. Medulla oblongata d. Midbrain

B The vomiting center of the brain lies in the medulla oblongata. The other locations listed are not related to vomiting.

When the right kidney is obstructed, how will the glomeruli and tubules in the left kidney compensate? a. Increase in number c. Develop collateral circulation b. Increase in size d. Increase speed of production

B These processes cause the contralateral (unobstructed) kidney to increase the size of individual glomeruli and tubules. The changes described by the other options are not correct.

What is the most common manifestation of portal hypertension-induced splenomegaly? a. Leukopenia c. Erythrocytopenia b. Thrombocytopenia d. Pancytopenia

B Thrombocytopenia (decreased platelet count) is the most common manifestation of congestive splenomegaly and can contribute to a tendency of increased bleeding.

Squamous cell carcinoma of the lung is best described as a tumor that causes which alterations? a. Abscesses and ectopic hormone production b. Airway obstruction and atelectasis c. Pleural effusion and shortness of breath d. Chest wall pain and early metastasis

B Typically, the tumors are centrally located near the hila and project into bronchi. Because of this central location, nonproductive cough or hemoptysis is common. Pneumonia and atelectasis are often associated with squamous cell carcinoma. Chest pain is a late symptom associated with large tumors. These tumors can remain fairly well localized and tend not to metastasize until late in the course of the disease. Squamous cell carcinomas are not associated with any of the other options.

Clinical manifestations that include irregular or heavy bleeding, the passage of large clots, and the depletion of iron stores support which diagnosis? a. Premenstrual syndrome c. Polycystic ovary syndrome b. Dysfunctional uterine bleeding d. Primary dysmenorrhea

B Unpredictable and variable bleeding, in terms of amount and duration, characterize dysfunctional uterine bleeding. Especially during perimenopause, dysfunctional bleeding also may involve flooding and the passage of large clots, which often indicate excessive blood loss. Excessive bleeding can lead to iron-deficiency anemia. This option is the only answer that demonstrates the clinical manifestations described.

When does urine formation and excretion begin? a. At birth c. By 6 months' gestation b. By 3 months' gestation d. By 8 months' gestation

B Urine formation and excretion begin by the third month of gestation.

Which statements characterize albumin? (Select all that apply.) a. Retains sodium to maintain water balance. b. Provides colloid osmotic pressure. c. Is synthesized in the liver. d. Is a carrier for drugs that have low water solubility. e. Is a small molecule

B, C, D Albumin is a plasma protein produced by the liver. It serves as a carrier molecule for the normal components of blood, as well as for drugs that have low solubility in water (e.g., free fatty acids, lipid-soluble hormones, thyroid hormones, bile salts). Albumin molecules are large and do not diffuse freely through the vascular endothelium, thus they maintain the critical colloidal osmotic pressure (or oncotic pressure) that regulates the passage of water and solutes into the surrounding tissues

Which statements are true concerning the sexually transmitted infection (STI) lymphogranuloma venereum? (Select all that apply.) a. Lymphogranuloma venereum is an STI commonly diagnosed in the United States. b. It begins as a skin infection. c. Lymphogranuloma venereum spreads to lymph tissues. d. Primary and secondary lesions are apparent with this STI. e. A 60-day course of oral erythromycin is the recommended treatment.

B, C, D Lymphogranuloma venereum is a chronic STI uncommon in the United States. The lesion begins as a skin infection and spreads to the lymph tissue, causing inflammation, necrosis, buboes, and abscesses of the inguinal lymph nodes. Primary lesions appear on the penis and scrotum in men and on the cervix, vaginal wall, and labia in women. Secondary lesions involve inflammation and swelling of the lymph nodes with the formation of large blue buboes that rupture and form draining ulcerative lesions. A 21-day or longer course of oral doxycycline or erythromycin is needed for treatment. Treatment of sexual partners is recommended.

Prerenal injury from poor perfusion can result from which condition? (Select all that apply.) a. Bilateral ureteral obstruction b. Renal vasoconstriction c. Renal artery thrombosis d. Hemorrhage e. Hypotension

B, C, D, E Poor perfusion can result from renal artery thrombosis, hypotension related to hypovolemia (dehydration, diarrhea, fluid shifts) or hemorrhage, renal vasoconstriction and alterations in renal regional blood flow, microthrombi, or kidney edema that restricts arterial blood flow. Bilateral ureteral obstruction is not associated with prerenal injuries.

Which statements are true regarding prostate cancer? (Select all that apply.) a. It ranks second to lung cancer as being most common among American men. b. A familial history of prostate cancer is a risk factor. c. Dietary habits seem to play a role in its development. d. African-American men have an increased risk for its development. e. Being over 65 years of age increases the risk for developing prostate cancer.

B, C, D, E Prostate cancer is the most commonly diagnosed non-skin cancer in American men, and the incidence varies greatly worldwide. Possible causes include a genetic predisposition, environmental and dietary factors, inflammation, and alterations in levels of hormones (e.g., testosterone, dihydrotestosterone, estradiol) and growth factors. Incidence is greatest among northwestern European and North American men (particularly African Americans) older than 65 years of age.

Which statements are true regarding renal colic? (Select all that apply.) a. Renal colic results in mild-to-moderate pain. b. Pain originates in the flank area. c. Renal colic indicates the presence of renal stones. d. Pain radiated to the groin. e. Renal colic indicates obstruction of the renal pelvis or proximal ureter.

B, C, D, E Renal colic is described as moderate-to-severe pain often originating in the flank and radiating to the groin. It usually indicates obstruction by renal stones of the renal pelvis or proximal ureter.

Which statement is true regarding alveoli? (Select all that apply.) a. The number of functioning alveoli is determined by birth. b. The alveoli begin to increase in size starting at 8 years of age. c. The complexity of the alveoli increases into adulthood. d. These structures produce surfactant. e. Capillaries are the origin of alveoli.

B, C, E Capillaries grow into the distal respiratory units that keep subdividing (alveolarization) to maximize the surface area for gas exchange. The number of alveoli continues to increase during the first 5 to 8 years of life, after which the alveoli increase in size and complexity. Surfactant is a lipid-protein mix that is produced by type II alveolar cells.

What are the common clinical manifestations of endometriosis? (Select all that apply.) a. Back and flank pain b. Infertility c. Dysuria d. Amenorrhea e. Dysmenorrhea

B, E Common clinical manifestations primarily include infertility, dysmenorrhea, dyschezia (pain on defecation), and dyspareunia (pain on intercourse).

Dysfunctional uterine bleeding (DUB), secondary to ovarian dysfunction, is abnormal uterine bleeding resulting from: (Select all that apply.) a. Endometriosis b. Progesterone deficiency c. Sexually transmitted infections d. Congenital abnormalities in the uterine structure e. Estrogen excess

B, E Of the options available, DUB, secondary to ovarian dysfunction, is a result of either progesterone deficiency or unopposed estrogen excess.

How are glucose and insulin used to treat hyperkalemia associated with acute renal failure? a. Glucose has an osmotic effect, which attracts water and sodium, resulting in more dilute blood and a lower potassium concentration. b. When insulin transports glucose into the cell, it also carries potassium with it. c. Potassium attaches to receptors on the cell membrane of glucose and is carried into the cell. d. Increasing insulin causes ketoacidosis, which causes potassium to move into the cell in exchange for hydrogen.

B. This selection is the only option that accurately describes glucose metabolism, causing potassium to move to the intracellular fluid; insulin infusions therefore can be effective in shifting potassium from the extracellular to intracellular space, along with the transport of glucose.

Hypercalciuria is primarily attributable to which alteration? a. Defective renal calcium reabsorption b. Intestinal hyperabsorption of dietary calcium c. Bone demineralization caused by prolonged immobilization d. Hyperparathyroidism

B. Hypercalciuria is usually attributable to intestinal hyperabsorption of dietary calcium and less commonly to a defect in renal calcium reabsorption. Hyperparathyroidism and bone demineralization associated with prolonged immobilization are also known to cause hypercalciuria but too a much lesser degree.

During which stage of syphilis do bloodborne bacteria spread to all the major organ systems? a. Primary c. Latent b. Secondary d. Tertiary

Bloodborne bacteria spread to all major organ systems during only stage II, secondary syphilis

When a woman's uterus is assessed as protruding through the entrance of the vagina to the hymen, which grade of prolapse does this indicate? a. 0 c. 2 b. 1 d. 3

C A grade 2 prolapse reaches the hymen (see Figure 24-10).

A peptic ulcer may occur in all of the following areas except the: a. Stomach c. Jejunum b. Duodenum d. Esophagus

C A peptic ulcer is a break, or ulceration, in the protective mucosal lining of the lower esophagus, stomach, or duodenum. This type of ulcer is not associated with the jejunum.

Considering the mediating factors of premenstrual syndrome (PMS), which medication may be used either continually or only during the menstrual period as a treatment for the condition? a. NSAIDs c. SSRIs b. Estrogen d. Progesterone

C A selective serotonin reuptake inhibitors (SSRI) (an antidepressant) relieves symptoms in approximately 60% to 90% of women and may be continually administered or only prescribed during the premenstrual period. Oral contraceptive pills that contain estrogen and progesterone also can be continuously used for up to 3 months to decrease the frequency of menstrual periods, PMS, and premenstrual dysphoric disorder (PMDD). Nonsteriodal antiinflammatory drugs (NSAIDs) would not be continually administered.

What are the abnormalities in cytokines found in children with cystic fibrosis (CF)? a. Deficit of interleukin (IL)-1 and an excess of IL-4, IL-12, and interferon-alpha (IFN-α) b. Deficit of IL-6 and an excess of IL-2, IL-8, and granulocyte colony-stimulating factor (G-CSF) c. Deficit of IL-10 and an excess of IL-1, IL-8, and TNF-α d. Deficit of IL-3 and an excess of IL-14, IL-24, and colony-stimulating factor (CSF)

C Abnormal cytokine profiles have been documented in CF airway fluids, including deficient IL-10 and excessive IL-1, IL-8, and TNF-α, all changes conducive to promoting inflammation. This selection is the only option that accurately identifies the abnormalities in cytokines observed in children with CF.

The collapse of lung tissue caused by the lack of collateral ventilation through the pores of Kohn is referred to as what type of atelectasis? a. Compression c. Absorption b. Perfusion d. Hypoventilation

C Absorption atelectasis is a result of the gradual absorption of air from obstructed or hypoventilated alveoli or from inhalation of concentrated oxygen or anesthetic agents. The other forms of atelectasis are not a result of the described mechanism.

Which antibiotics are considered "major culprits" in causing nephrotoxic acute tubular necrosis (ATN)? a. Penicillin and ampicillin c. Gentamicin and tobramycin b. Vancomycin and bacitracin d. Cefazolin and cefepime

C Although numerous antibiotics can produce nephrotoxic ATN, the aminoglycosides (gentamicin, tobramycin) are the major culprits.

Where in the respiratory tract do the majority of foreign objects aspirated by children finally lodge? a. Trachea c. Bronchus b. Left lung d. Bronchioles

C Approximately 75% of aspirated foreign bodies lodge in a bronchus. The other options are not locations where children aspirate the majority of foreign objects.

What is the life span of an erythrocyte (in days)? a. 20 to 30 c. 100 to 120 b. 60 to 90 d. 200 to 240

C Because it cannot undergo mitotic division, the erythrocyte has a limited life span of approximately 120 days

Which mineral accounts for the most common type of renal stone? a. Magnesium-ammonium-phosphate c. Calcium oxalate b. Uric acid d. Magnesium phosphate

C Calcium stones (calcium phosphate or calcium oxalate) account for 70% to 80% of all stones requiring treatment.

What is the initial step in the management of emphysema? a. Inhaled anticholinergic agents c. Cessation of smoking b. Beta agonists d. Surgical reduction of lung volume

C Chronic management of emphysema begins with smoking cessation. Pharmacologic management includes inhaled anticholinergic agents, and beta agonists should be prescribed. Pulmonary rehabilitation, improved nutrition, and breathing techniques all can improve symptoms. Oxygen therapy is indicated in chronic hypoxemia but must be administered with care. In selected patients, lung volume reduction surgery or transplantation can be considered.

Which nutrients are necessary for the synthesis of DNA and the maturation of erythrocytes? a. Protein and niacin c. Cobalamin (vitamin B12) and folate b. Iron and vitamin B6 (pyridoxine) d. Pantothenic acid and vitamin C

C Cobalamin and folate are necessary for the synthesis of DNA and for the maturation of erythrocytes. The remaining options are not necessary for these processes to occur

Which criterion is used to confirm a diagnosis of asthma in an 8-year-old child? a. Parental history of asthma b. Serum testing that confirms increased immunoglobulin E (IgE) and eosinophil levels c. Reduced expiratory flow rates confirmed by spirometry testing d. Improvement on a trial of asthma medication

C Confirmation of the diagnosis of asthma relies on pulmonary function testing using spirometry, which can be accomplished only after the child is 5 to 6 years of age. Reduced expiratory flow rates that are reversible in response to an inhaled bronchodilator would be characteristic abnormalities. For younger children, an empiric trial of asthma medications is commonly initiated. The remaining options are major historical and physical factors that contribute but do not confirm the diagnosis of asthma in children.

Which factor contributes to the production of mucus associated with chronic bronchitis? a. Airway injury c. Increased Goblet cell size b. Pulmonary infection d. Bronchospasms

C Continual bronchial inflammation causes bronchial edema and increases the size and number of mucous glands and goblet cells in the airway epithelium. Thick, tenacious mucus is produced and cannot be cleared because of impaired ciliary function (see Figure 35-13). The lung's defense mechanisms are therefore compromised, increasing a susceptibility to pulmonary infection, which contributes to airway injury. Frequent infectious exacerbations are complicated by bronchospasm with dyspnea and productive cough

Creatinine is primarily excreted by glomerular filtration after being constantly released from what type of tissue? a. Nervous system c. Muscle b. Kidneys d. Liver

C Creatinine is constantly released from only muscle tissue to be excreted by glomerular filtration.

Which statement best describes cystic fibrosis? a. Obstructive airway disease characterized by reversible airflow obstruction, bronchial hyperreactivity, and inflammation b. Respiratory disease characterized by severe hypoxemia, decreased pulmonary compliance, and diffuse densities on chest x-ray imaging c. Pulmonary disorder involving an abnormal expression of a protein-producing viscous mucus that obstructs the airways, pancreas, sweat ducts, and vas deferens d. Pulmonary disorder characterized by atelectasis and increased pulmonary resistance as a result of a surfactant deficiency

C Cystic fibrosis is best described as a pulmonary disorder involving an abnormal expression of a protein-producing viscous mucus that obstructs the airways, pancreas, sweat ducts, and vas deferens. This selection is the only option that accurately describes cystic fibrosis.

Which term is used to identify the descent of the posterior bladder and trigone into the vaginal canal? a. Rectocele c. Cystocele b. Vaginocele d. Enterocele

C Cystocele is the only term used to identify the descent of a portion of the posterior bladder wall and trigone into the vaginal canal; the trauma of childbirth is usually the cause.

Fluid-filled squishy sacs characterize which breast disorder? a. Paget disease c. Nonproliferative breast lesions b. Cysts d. Lobular carcinoma in situ

C Cysts (fluid-filled sacs) are a specific type of lump that commonly occurs in women in their 30s, 40s, and early 50s. Cysts feel squishy when they occur close to the surface of the breast; however, when deeply embedded, cysts can feel hard. The other options do not accurately identify the disorder associated with these symptoms.

During the latent period of a herpes virus infection, where in the host cell is the genome of the virus maintained? a. Mitochondria c. Nucleus b. Lysosomes d. Cytoplasm

C During the latent period, the genome for the virus is maintained in the host cell nucleus without causing the death of the cell.

What medical term is used to identify the presence of pus in the pleural space? a. Plural effusion c. Empyema b. Asthma d. Pneumonia

C Empyema is the presence of pus in the pleural space. This condition is not identified by any of the other options.

How does an established gonococcal infection usually express itself in newborns? a. Generalized skin rash 4 to 6 days after birth b. Systemic infection with fever c. Bilateral corneal ulceration d. Yellow vaginal or penile discharge approximately 10 days after birth

C Established infection causes bilateral corneal ulceration. The other options are not usual sites for such an infection.

Which of the following causes condylomata acuminata or genital warts? a. Chlamydia c. Human papillomavirus (HPV) b. Adenovirus d. Herpes simplex virus 1 (HSV-1)

C Genital warts are quite contagious and are a result of only HPV.

Which disorder is characterized by damage to the mucosa of the duodenum and jejunum and impaired secretion of secretin, cholecystokinin, and pancreatic enzymes? a. Wilson disease c. Gluten-sensitive enteropathy b. Cystic fibrosis d. Galactosemia

C Gluten-sensitive enteropathy is characterized by damage to the mucosa of the duodenum and jejunum and has secondary effects that exacerbate malabsorption. The secretion of intestinal hormones, such as secretin and cholecystokinin, may be diminished. Because these chemical messengers are scarce, secretion of pancreatic enzymes and expulsion of bile from the gallbladder decrease. These statements are not true of the other options.

Which statement is true regarding ventilation? a. Hypoventilation causes hypocapnia. b. Hyperventilation causes hypercapnia. c. Hyperventilation causes hypocapnia. d. Hyperventilation results in an increased partial pressure of arterial carbon dioxide (PaCO2).

C Hyperventilation is alveolar ventilation that exceeds metabolic demands. The lungs remove carbon dioxide at a faster rate than produced by cellular metabolism, resulting in decreased PaCO2 or hypocapnia. None of the remaining options are accurate statements.

In which type of pleural effusion does the fluid become watery and diffuse out of the capillaries as a result of increased blood pressure or decreased capillary oncotic pressure? a. Exudative c. Transudative b. Purulent d. Large

C In transudative pleural effusion, the fluid, or transudate, is watery and diffuses out of the capillaries as a result of disorders that increase intravascular hydrostatic pressure or decrease capillary oncotic pressure. The described mechanism is not associated with the other forms of pleural effusion.

Which immunoglobulin (Ig) is present in childhood asthma? a. IgM c. IgE b. IgG d. IgA

C Included in the long list of asthma-associated genes are those that code for increased levels of immune and inflammatory mediators (e.g., interleukin [IL]-4, IgE, leukotrienes), nitric oxide, and transmembrane proteins in the endoplasmic reticulum. None of the other options are associated with childhood asthma.

What is the primary cause of peptic ulcers? a. Hypersecretion of gastric acid c. Helicobacter pylori b. Hyposecretion of pepsin d. Escherichia coli

C Infection with H. pylori is a primary cause of peptic ulcers.

Which term is used to describe an intestinal obstruction caused by the invagination of the ileum into the cecum and part of the ascending colon by collapsing through the ileocecal valve? a. Congenital aganglionic megacolon c. Intussusception b. Malrotation d. Volvulus

C Intussusception is the telescoping or invagination of one portion of the intestine into another section of intestine. Usually, the ileum invaginates the cecum and part of the ascending colon by collapsing through the ileocecal valve. The other terms are not used to describe this event.

Which term is used to identify benign uterine tumors that develop from smooth muscle cells in the myometrium and are commonly called uterine fibroids? a. Endometrial polyps c. Leiomyomas b. Myometrial polyps d. Myometriomas

C Leiomyomas, commonly called myomas or uterine fibroids, are benign smooth muscle tumors in the myometrium (see Figure 24-14). The other terms do not accurately identify the tumors described.

In acute respiratory distress syndrome (ARDS), alveoli and respiratory bronchioles fill with fluid as a result of which mechanism? a. Compression on the pores of Kohn, thus preventing collateral ventilation b. Increased capillary permeability, which causes alveoli and respiratory bronchioles to fill with fluid c. Inactivation of surfactant and the impairment of type II alveolar cells d. Increased capillary hydrostatic pressure that forces fluid into the alveoli and respiratory bronchioles

C Lung inflammation and injury damage the alveolar epithelium and the vascular endothelium. Surfactant is inactivated, and its production by type II alveolar cells is impaired as alveoli and respiratory bronchioles fill with fluid or collapse. The other options would not trigger the described response.

Detrusor hyperreflexia develops from neurologic disorders that originate where? a. Spinal cord between C2 and S1 c. Above the pontine micturition center b. Spinal cord between S2 and S4 d. Below the cauda equina

C Neurologic disorders that develop above the pontine micturition center result in detrusor hyperreflexia, also known as an uninhibited or reflex bladder. This selection is the only option responsible for detrusor hyperreflexia.

Considering the normal frequency of bowel evacuation, how infrequently can evacuation occur and still be considered within normal range? a. Once a day c. Once a week b. Once every 2 days d. Once every 2 weeks

C Normal bowel habits range from two or three evacuations per day to one per week.

incomplete fusion of the nasomedial and intermaxillary process during the fourth week of embryonic development causes which condition in an infant? a. Cleft palate c. Cleft lip b. Sinus dysfunction d. Esophageal malformation

C Of the available options, only a cleft lip is caused by the incomplete fusion of the nasomedial and intermaxillary process during the fourth week of embryonic development.

What is the most common infectious cause of orchitis and one that usually affects postpubertal boys? a. Herpes c. Mumps b. Escherichia coli d. Cytomegalovirus

C Of the options available, mumps is the most common infectious cause of orchitis and usually affects postpubertal boys.

What are the cardinal symptoms of small intestinal obstruction? a. Constant, dull pain in the lower abdomen relieved by defecation b. Acute, intermittent pain 30 minutes to 2 hours after eating c. Colicky pain caused by distention, followed by vomiting d. Excruciating pain in the hypogastric area caused by ischemia

C Of the options available, only colicky pain caused by distention followed by vomiting are considered the cardinal symptoms of a small intestinal obstruction.

At 2 or 3 weeks of age, an infant who has been well fed and has gained weight begins to vomit for no apparent reason. The vomiting gradually becomes more forceful. These symptoms may be indicative of which disorder? a. Esophageal atresia c. Pyloric stenosis b. Congenital aganglionic megacolon d. Galactosemia

C Of the options available, only the clinical manifestations of pyloric stenosis—an infant who has been well fed and has gained weight begins to vomit without an apparent reason—can appear between 2 and 3 weeks after birth. The vomiting gradually becomes more forceful.

Priapism has been associated with the abuse of what substance? a. Marijuana c. Cocaine b. Alcohol d. Heroin

C Of the options available, priapism has been associated with cocaine use.

Bladder cancer is associated with the gene mutation of which gene? a. c-erbB2 b. Human epidermal growth factor receptor 2 (HER2) c. TP53 d. myc

C Oncogenes of the ras gene family and tumor-suppressor genes including TP53 mutations and the inactivation of the retinoblastoma gene (pRb) are implicated in bladder cancer. This process is not associated with the other options.

What are typical findings on breast palpation of a woman diagnosed with simple fibroadenoma? a. Painful, round, movable, and fluid-filled mass b. Painless, movable, hard, and irregular mass c. Smooth, solid, mobile, and well-circumscribed mass d. Smooth, nonmovable, irregular, and soft mass

C Only this option accurately identifies the typical palpation results of a simple fibroadenoma.

What statement concerning the pathogenetic mechanisms of polycystic ovarian syndrome (POS) is true? a. POS causes a decrease in leptin levels; this decrease reduces the hypothalamic pulsatility of gonadotropin-releasing hormone, which reduces the number of follicles that mature. b. POS is a result of a disorder in the anterior pituitary that increases the follicle-stimulating hormone, which reduces the luteinizing hormone released. c. POS is a result of a combination of conditions that include oligo-ovulation or anovulation, elevated levels of androgens, or clinical signs of hyperandrogenism and polycystic ovaries. d. POS inhibits testosterone, which stimulates androgen secretion by the ovarian stroma and indirectly reduces sex hormone-binding globulin.

C POS has at least two of the following conditions: oligo-ovulation or anovulation, elevated levels of androgens, or clinical signs of hyperandrogenism and polycystic ovaries. Of the options available, only this answer accurately defines the pathogenetic mechanisms of POS.

Clinical manifestations of inspiratory crackles, increased tactile fremitus, egophony, and whispered pectoriloquy are indicative of which respiratory condition? a. Chronic bronchitis c. Pneumonia b. Emphysema d. Asthma

C Physical examination may reveal signs of pulmonary consolidation, such as inspiratory crackles, increased tactile fremitus, egophony, and whispered pectoriloquy, which support a diagnosis of pneumonia. The presentations of the other options are not consistent with the described symptoms.

What is the role of thromboxane A (TXA2) in the secretion stage of hemostasis? a. Stimulates the synthesis of serotonin. b. Promotes vasodilation. c. Stimulates platelet aggregation. d. Promotes formation of cyclooxygenase.

C Platelet aggregation is primarily stimulated by TXA2 and adenosine diphosphate (ADP), which induce functional fibrinogen receptors on the platelet. The other options do not present an accurate description of the role of thromboxane A

Besides dyspnea, what is the most common characteristic associated with pulmonary disease? a. Chest pain c. Cough b. Digit clubbing d. Hemoptysis

C Pulmonary disease is associated with many signs and symptoms, and their specific characteristics often help in identifying the underlying disorder. The most common characteristics are dyspnea and cough. Others include abnormal sputum, hemoptysis, altered breathing patterns, hypoventilation and hyperventilation, cyanosis, clubbing of the digits, and chest pain.

What is the primary cause of respiratory distress syndrome (RDS) of the newborn? a. Immature immune system c. Surfactant deficiency b. Small alveoli d. Anemia

C RDS is primarily caused by surfactant deficiency and secondarily by a deficiency in alveolar surface area for gas exchange. None of the other options are related to the cause of RDS.

What is the chief predisposing factor for respiratory distress syndrome (RDS) of the newborn? a. Low birth weight b. Alcohol consumption during pregnancy c. Premature birth d. Smoking during pregnancy

C RDS of the newborn, also known as hyaline membrane disease (HMD), is a major cause of morbidity and mortality in premature newborns. None of the other options are considered the chief predisposing factors for RDS.

Which statement concerning erythrocytes is true? a. Erythrocytes contain a nucleus, mitochondria, and ribosomes. b. Erythrocytes synthesize proteins. c. Erythrocytes have the ability to change shape to squeeze through microcirculation. d. Erythrocyte colony-stimulating factor (E-CSF) stimulates erythrocytes.

C Reversible deformity enables the erythrocyte to assume a more compact torpedo-like shape, squeeze through the microcirculation, and return to normal. The other options are not accurate statements about erythrocytes.

In children, the risk factors for hepatitis B virus (HBV) are primarily associated with: a. Living in urban communities b. Mothers who are hepatitis C carriers c. Transfusion therapy for hemophilia d. Those of Hispanic ethnic background

C Risk factors for HBV infection include infants of mothers who are carriers of chronic hepatitis B surface antigen (HBsAg), hemophiliacs who receive frequent blood transfusions, children who abuse parenteral drugs, and children who live in residences for those who are mentally delayed. HBV is endemic in China and other parts of Asia where most infections occur in infants and children as a result of maternal-neonatal transmission.

What is the cause of faulty digestion of fats in those diagnosed with cystic fibrosis? a. Bile ducts obstructed with mucus, prohibiting the release of bile b. Failure to metabolize fat-soluble vitamins c. Deficiency of pancreatic enzymes d. Fat malabsorption that now occurs in the jejunum

C Severe problems with maldigestion of proteins, carbohydrates, and fats occur because of the insufficient secretion of pancreatic enzymes. This selection is the only option that accurately describes why fatty stools are observed in individuals diagnosed with cystic fibrosis.

What is the consequence of a splenectomy? a. The level of iron in circulation increases. b. Antibody production increases to improve immune function. c. The number of defective cells in circulation increases. d. The number of clotting factors increases.

C Splenic absence from any cause (e.g., atrophy, traumatic injury, removal because of disease) has several secondary effects on the body, among them an increase in morphologically defective blood cells in the circulation, confirming the spleen's role in removing old or damaged cells. This description of the consequence of a splenectomy is not accurate for the other options.

What is the most common clinical sign of portal hypertension in children? a. Right heart failure c. Splenomegaly b. Pulmonary edema d. Diarrhea

C Splenomegaly is the most common sign of portal hypertension in children.

Which statement is false regarding the sources of increased ammonia that contribute to hepatic encephalopathy? a. End products of intestinal protein digestion are sources of increased ammonia. b. Digested blood leaking from ruptured varices is a source of increased ammonia. c. Accumulation of short-chain fatty acids that is attached to ammonia is a source of increased ammonia. d. Ammonia-forming bacteria in the colon are sources of increased ammonia.

C The accumulation of short-chain fatty acids, serotonin, tryptophan, and false neurotransmitters probably contributes to neural derangement and is not associated with ammonia levels. The other options provide accurate information regarding how the sources of ammonia contribute to hepatic encephalopathy.

The adult intestine processes approximately how many liters of luminal content per day? a. 3 c. 9 b. 6 d. 12

C The adult intestine processes approximately 9 L of luminal content per day. Of this amount, 2 L is ingested and the remaining 7 L consists of intestinal secretions.

What is the life span of platelets (in days)? a. 10 c. 90 b. 30 d. 120

A A platelet circulates for approximately 10 days and ages. Macrophages of the mononuclear phagocyte system, mostly in the spleen, remove platelets

Which condition is a fulminant form of respiratory failure characterized by acute lung inflammation and diffuse alveolocapillary injury? a. Acute respiratory distress syndrome (ARDS) b. Pneumonia c. Pulmonary emboli d. Acute pulmonary edema

A ARDS is a fulminant form of respiratory failure characterized by acute lung inflammation and diffuse alveolocapillary injury. The described pathologic characteristics are not associated with the other options.

What local complication of a gonococcal infection is diagnosed in approximately 10% of affected women? a. Acute salpingitis c. Vaginitis b. Cystitis d. Cervicitis

A Acute salpingitis, or pelvic inflammatory disease (PID), is the most common local complication in women. Approximately 10% of women with untreated cervical gonorrhea develop PID.

When does an individual have their full complement of renal nephrons? a. At birth c. At puberty b. At 6 months of age d. Between the ages 18 and 21 years

A All the nephrons are present at birth, and their number does not increase as the kidney grows and matures.

Respirations that are characterized by alternating periods of deep and shallow breathing are a result of which respiratory mechanism? a. Decreased blood flow to the medulla oblongata b. Increased partial pressure of arterial carbon dioxide (PaCO2), decreased pH, and decreased partial pressure of arterial oxygen (PaO2) c. Stimulation of stretch or J-receptors d. Fatigue of the intercostal muscles and diaphragm

A Alternating periods of deep and shallow breathing are characteristic of Cheyne-Stokes respirations and are the result of any condition that slows the blood flow to the brainstem, which in turn slows impulses that send information to the respiratory centers of the brainstem. None of the remaining options are responsible for the described breathing pattern.

Which drug may be prescribed orally for outbreak management of herpes simplex viral (HSV) infections? a. Acyclovir (Zovirax) c. Zidovudine (AZT) (Retrovir) b. 5-Fluorouracil (5-FU) d. Bichloroacetic acid (BCA)

A Although no curative treatment for HSV infection is known, only oral acyclovir, valacyclovir, penciclovir, and famciclovir are used for primary and periodic outbreaks and to prevent recurrences.

Goodpasture syndrome is an example of which of the following? a. Antiglomerular basement membrane disease b. Acute glomerulonephritis c. Chronic glomerulonephritis d. Immunoglobulin A (IgA) nephropathy

A Antiglomerular basement membrane disease (Goodpasture syndrome) is associated with immunoglobulin G (IgG) antibody formation against pulmonary capillary and glomerular basement membranes. Goodpasture syndrome is not an example of any of the other options.

What causes a person with cystic fibrosis to experience an exocrine pancreatic insufficiency? a. Pancreatic ducts are obstructed with mucus. b. Impaired blood supply to the pancreas causes ischemia. c. A genetically impaired pancreas is unable to produce digestive enzymes. d. The pancreas has a volvulus at the ampulla of Vater.

A Approximately 85% of individuals with cystic fibrosis experience pancreatic insufficiency. Obstruction of the pancreatic ducts with thick mucus blocks the flow of pancreatic enzymes and causes degenerative and fibrotic changes in the pancreas. This selection is the only option that accurately describes why an exocrine pancreatic insufficiency is observed in individuals diagnosed with cystic fibrosis.

Which of the following is the most common route of lower respiratory tract infection? a. Aspiration of oropharyngeal secretions b. Inhalation of microorganisms c. Microorganisms spread to the lung via blood d. Poor mucous membrane protection

A Aspiration of oropharyngeal secretions is the most common route of lower respiratory tract infection; thus the nasopharynx and oropharynx constitute the first line of defense for most infectious agents. The other options are not common routes of lower respiratory tract infections.

Which statement accurately describes childhood asthma? a. An obstructive airway disease characterized by reversible airflow obstruction, bronchial hyperreactivity, and inflammation b. A pulmonary disease characterized by severe hypoxemia, decreased pulmonary compliance, and diffuse densities on chest x-ray imaging c. A pulmonary disorder involving an abnormal expression of a protein, producing viscous mucus that lines the airways, pancreas, sweat ducts, and vas deferens d. An obstructive airway disease characterized by atelectasis and increased pulmonary resistance as a result of a surfactant deficiency

A Asthma is an obstructive airway disease characterized by reversible airflow obstruction, bronchial hyperreactivity, and inflammation. This selection is the only option that accurately describes childhood asthma.

Which statement is false concerning the pathophysiologic process of alcoholic cirrhosis? a. Inflammation and damage leading to cirrhosis begin in the bile canaliculi. b. Alcohol is transformed to acetaldehyde, which promotes liver fibrosis. c. Mitochondrial function is impaired, decreasing oxidation of fatty acids. d. Acetaldehyde inhibits export of proteins from the liver.

A Biliary cirrhosis differs from alcoholic cirrhosis in that the damage and inflammation leading to cirrhosis begin in bile canaliculi and bile ducts, rather than in the hepatocytes. The other options provide true information regarding the pathophysiologic process of alcoholic cirrhosis.

How does chest wall compliance in an infant differ from that of an adult? a. An adult's chest wall compliance is lower than an infant's. b. An adult's chest wall compliance is higher than an infant's. c. An adult's chest wall compliance is the same as an infant's. d. An adult's chest wall compliance is dissimilar to that of an infant's.

A Chest wall compliance is higher in infants than it is in adults, particularly in premature infants.

What is a cause of chronic antral gastritis? a. Helicobacter pylori bacteria b. Development of autoantibodies to gastric H+/K+ ATPase c. Pernicious anemia d. Reflux of bile and alkaline pancreatic secretions

A Chronic antral gastritis generally involves only the antrum and is more common than fundal gastritis. It is caused by H. pylori bacteria or the chronic use of alcohol, tobacco, and nonsteroidal antiinflammatory drugs. None of the other options are associated with the cause of chronic antral gastritis.

the most successful treatment for chronic asthma begins with which action? a. Avoidance of the causative agent b. Administration of broad-spectrum antibiotics c. Administration of drugs that reduce bronchospasm d. Administration of drugs that decrease airway inflammation

A Chronic management of asthma begins with the avoidance of allergens and other triggers. The effectiveness of the other options is reliant on the avoidance of triggers.

The drug heparin acts in hemostasis by which processes? a. Inhibiting thrombin and antithrombin III (AT-III) b. Preventing the conversion of prothrombin to thrombin c. Shortening the fibrin strands to retract the blood clot d. Degrading the fibrin within blood clots

A Clinically administered heparin or heparin sulfate (on the surface of endothelial cells) binds to AT-III and induces a conformational change that greatly enhances its activity. Under normal conditions, the presence of endothelial cell heparin sulfate and available AT-III in the circulation cooperate to protect the vessels from the effects of spontaneously activated thrombin. The other options do not accurately describe the role heparin plays in hemostasis.

Considering the pathophysiologic characteristics of primary amenorrhea, what anatomic structure is involved in compartment II? a. Ovary c. Hypothalamus b. Anterior pituitary d. Vagina

A Compartment II disorders involve only the ovary.

Cystic fibrosis is caused by which process? a. Autosomal recessive inheritance c. Infection b. Autosomal dominant inheritance d. Malignancy

A Cystic fibrosis is an autosomal recessive inherited disorder that is associated with defective epithelial ion transport. None of the other options cause cystic fibrosis.

What factor associated with gluten-sensitive enteropathy (celiac sprue) causes an infant to bruise and bleed easily? a. Vitamin K deficiency from fat malabsorption b. Bone marrow function depression c. Iron, folate, and B12 deficiency anemias d. Prescribed daily warfarin (Coumadin)

A Deficiencies of fat-soluble vitamins (such as vitamin K) are common in children with gluten-sensitive enteropathy. Vitamin K malabsorption leads to hypoprothrombinemia, causing the child to bruise and bleed easily. This selection is the only option that accurately describes the mechanism that causes bruising and bleeding in children diagnosed with celiac sprue.

Which symptom is characteristic of bulimia nervosa? a. Recurrent episodes of binge eating with fears of not being able to stop eating. b. Fear of becoming obese, despite progressive weight loss. c. Perception that the body is fat when it is actually underweight. d. Absence of three consecutive menstrual periods.

A Diagnosis of bulimia is based on, among other findings, recurrent episodes of binge eating during which the individual fears not being able to stop. The remaining options are characteristic of anorexia nervosa.

Which abnormal laboratory value is found in glomerular disorders? a. Elevated creatinine concentration c. Elevated immunoglobulin A (IgA) b. Low blood urea nitrogen (BUN) d. Low serum complement

A Elevated creatinine concentration is an abnormal laboratory value found in glomerular disorders. Reduced glomerular filtration rate during glomerular disease is evidenced by elevated plasma urea, creatinine concentration, or reduced renal creatinine clearance. Glomerular disorders are not associated with the other options.

What is the function of erythrocytes? a. Tissue oxygenation c. Infection control b. Hemostasis d. Allergy response

A Erythrocytes are solely responsible for tissue oxygenation.

Cystic fibrosis is characterized by which symptom? a. Excessive mucus production c. Low sodium content in perspiration b. Elevated blood glucose levels d. Abnormally thin exocrine secretions

A Excessive mucus production characterizes cystic fibrosis. However, the pathophysiologic triad that is the hallmark of cystic fibrosis includes (1) pancreatic enzyme deficiency, which causes maldigestion; (2) overproduction of mucus in the respiratory tract and an inability to clear secretions, which cause progressive chronic obstructive pulmonary disease; and (3) abnormally elevated sodium and chloride concentrations in sweat. Exocrine secretions tend to be abnormally thick and precipitate in the glandular ducts, obstructing flow. An elevated blood glucose level is not associated with this disorder.

What term is used to identify the condition that exists when the urethral meatus is located on the undersurface of the penis? a. Hypospadias c. Hyperspadias b. Epispadias d. Chordee

A Hypospadias is a congenital condition in which the urethral meatus is located on the ventral side or undersurface of the penis. This is the only term used for this condition.

When a patient has small, vesicular lesions that last between 10 and 20 days, which sexually transmitted infection is suspected? a. Genital herpes c. Syphilis b. Chancroid d. Chlamydia

A If symptoms occur, the individual may have small (1 to 2 mm), multiple, vesicular lesions that are generally located on the labia minora, fourchette, or penis. They may also appear on the cervix, buttocks, and thighs and are often painful and pruritic. These lesions usually last approximately 10 to 20 days. The other options do not demonstrate these symptoms.

In women, what is the usual site of original gonococcal infection? a. Endocervical canal c. Fallopian tube b. Vagina d. Labia majora

A In women, the endocervical canal (inner portion of the cervix) is the usual site of original gonococcal infection, although urethral colonization and infection of Skene or Bartholin glands also are common. The other options are not usually associated with gonococcal infections.

Why is nasal congestion a serious threat to young infants? a. Infants are obligatory nose breathers. b. Their noses are small in diameter. c. Infants become dehydrated when mouth breathing. d. Their epiglottis is proportionally greater than the epiglottis of an adult's.

A Infants up to 2 to 3 months of age are obligatory nose breathers and are unable to breathe in through their mouths. Nasal congestion is therefore a serious threat to a young infant. This selection is the only option that accurately describes why nasal congestion is a serious threat to young infants.

Which virus is a precursor for developing cervical intraepithelial neoplasia (CIN) and cervical cancer? a. Human papillomavirus (HPV) c. Herpes simplex II virus (HSV) b. Epstein-Barr virus (EBV) d. Cytomegalovirus (CMV)

A Infection with high-risk (oncogenic) types of HPV (predominantly 16 and 18) is a necessary precursor to the development of precancerous dysplasia of the cervix that leads to invasive cancer. The other options are not precursors to CIN and cervical cancer.

Which term is used to identify a condition in which the developing colon remains in the upper right quadrant instead of moving to its normal location? a. Intestinal malrotation c. Duodenal obstruction b. Ileocecal displacement d. Pyloric stenosis

A Intestinal malrotation is the only term used to identify a condition in which rotation does not occur and the colon remains in the upper right quadrant, where an abnormal membrane may press on and obstruct the duodenum.

By what mechanism does intussusception cause an intestinal obstruction? a. Telescoping of part of the intestine into another section of intestine, usually causing strangulation of the blood supply b. Twisting the intestine on its mesenteric pedicle, causing occlusion of the blood supply c. Loss of peristaltic motor activity in the intestine, causing an adynamic ileus d. Forming fibrin and scar tissue that attach to the intestinal omentum, causing obstruction

A Intussusception is the telescoping of part of the intestine into another section of intestine, usually causing strangulation of the blood supply. This selection is the only option that accurately describes how intussusception causes an intestinal obstruction.

Which type of precocious puberty causes the child to develop some secondary sex characteristics of the opposite sex? a. Mixed c. Isosexual b. Incomplete d. Homosexual

A Mixed precocious puberty, which is virilization of a girl or feminization of a boy, causes the child to develop some secondary sex characteristics of the opposite sex. This option is the only answer that accurately identifies the type of precocious puberty described.

Which statement is consistent with dumping syndrome? a. Dumping syndrome usually responds well to dietary management. b. It occurs 1 to 2 hours after eating. c. Constipation is often a result of the dumping syndrome. d. It can result in alkaline reflux gastritis.

A Most individuals with the dumping syndrome respond well to dietary management. None of the other options is associated with the dumping syndrome.

Which statement is false regarding the risk of transmission of the herpes simplex virus (HSV) from mother to fetus? a. Neonatal infection of HSV rarely occurs in the intrapartum or postpartum period. b. The risk is higher in women who have a primary HSV infection. c. The risk is higher in women who experience ruptured membranes more than 6 hours before delivery. d. The risk is higher when internal fetal monitoring devices are used

A Neonatal infections can occur in utero or, more commonly, during the intrapartum or postpartum period. The other options are accurate statements.

What medical term is used to identify a functional urinary tract obstruction caused by an interruption of the nerve supply to the bladder? a. Neurogenic bladder c. Necrotic bladder b. Obstructed bladder d. Retrograde bladder

A Neurogenic bladder is a general term for bladder dysfunction caused by neurologic disorders. The types of dysfunction are related to the sites in the nervous system that control sensory and motor bladder function (see Figure 38-3). None of the other options correctly identify the described condition.

Which blood cells are the chief phagocytes involved in the early inflammation process? a. Neutrophils c. Eosinophils b. Monocytes d. Erythrocytes

A Neutrophils are the chief phagocytes of early inflammation.

Which medication classification is generally included in the treatment of silicosis? a. Corticosteroids c. Bronchodilators b. Antibiotics d. Expectorants

A No specific treatment exists for silicosis, although corticosteroids may produce some improvement in the early, more acute stages. The other options are not generally prescribed.

Where are Langerhans cells found? a. Skin c. Kidney b. Intestinal lining d. Thyroid

A Of the available options, only the skin is the location for Langerhans cells

What are the clinical manifestations of testicular cancer? a. Firm, nontender testicular mass c. Painful fluid-filled testicular mass b. Painful, mobile, firm testicular mass d. Soft, nontender testicular mass

A Of the options available, a firm, painless testicular enlargement is commonly identified as the first sign of testicular cancer.

Which benign breast tumor affects postmenopausal women and is characterized by the principal lactiferous ducts becoming dilated and filled with cellular debris? a. Mammary duct ectasia c. Phyllodes tumor b. Intraductal papilloma d. Fibroadenoma

A Of the options available, only mammary duct ectasia is associated with the age and the identified pathologic characteristics

What is the first sign of puberty in girls? a. Breast enlargement c. Menstruation b. Growth of pubic hair d. Vaginal discharge

A Of the options available, the first sign of puberty in girls is usually thelarche or breast development.

Which clinical manifestations of a urinary tract infection may be demonstrated in an 85-year-old individual? a. Confusion and poorly localized abdominal discomfort b. Dysuria, frequency, and suprapubic pain c. Hematuria and flank pain d. Pyuria, urgency, and frequency

A Older adults with cystitis may demonstrate confusion or vague abdominal discomfort or otherwise be asymptomatic.

Blood cells that differentiate into macrophages are known as: a. Monocytes c. Eosinophils b. Neutrophils d. Basophils

A Only monocytes migrate into a variety of tissues and fully mature into tissue macrophages and myeloid dendritic cells

Which microorganism is sexually transmitted, primarily by homosexual men, through infected feces? a. Shigellosis c. Giardia lamblia b. Cytomegalovirus (CMV) d. Entamoeba histolytica

A Only the Shigella infection, termed shigellosis, is transmitted by contact with infected feces particularly among homosexual men.

What is the first indication of nephrotic syndrome in children? a. Periorbital edema c. Frothy urine b. Scrotal or labial edema d. Ascites

A Onset of nephritic syndrome is insidious, with periorbital edema as the first sign of the disorder. None of the other options represent the first indication of nephritic syndrome in children.

Which factor increases the risk for ovarian cancer after the age of 40 years? a. Use of fertility drugs c. Multiple pregnancies b. Oral contraceptive use d. Prolonged lactation

A Ovarian cancer in women older than 40 years of age is associated with early menarche, late menopause, nulliparity, and the use of fertility drugs. The other options are not necessarily related to women older than the age of 40 years.

Which age group should be targeted for testicular cancer education and screening? a. 15 to 35 year olds c. 30 to 55 year olds b. 20 to 45 year olds d. 45 to 70 year olds

A Overall, testicular cancers are rare, yet they are the most common form of cancer in young men between the ages of 15 and 35 years.

Pulmonary artery hypertension (PAH) results from which alteration? a. Narrowed pulmonary capillaries c. Destruction of alveoli b. Narrowed bronchi and bronchioles d. Ischemia of the myocardium

A PAH is characterized by endothelial dysfunction with an overproduction of vasoconstrictors (e.g., thromboxane, endothelin) and decreased production of vasodilators (e.g., nitric oxide, prostacyclin), resulting in narrowed pulmonary capillaries. None of the remaining options result in pulmonary hypertension.

How can abdominal pain that is visceral in nature best be described? a. Abdominal pain that is visceral in nature is diffused, vague, poorly localized, and dull. b. It travels from a specific organ to the spinal cord. c. The pain lateralizes from only one side of the nervous system. d. Abdominal pain is associated with the peristalsis of the gastrointestinal tract.

A Pain is usually felt near the midline in the epigastrium (upper midabdomen), midabdomen, or lower abdomen. The pain is poorly localized, is dull rather than sharp, and is difficult to describe. None of the other options accurately describe this type of pain.

Fluid in the pleural space characterizes which condition? a. Pleural effusion c. Bronchiectasis b. Atelectasis d. Ischemia

A Pleural effusion is the presence of fluid in the pleural space. This condition is not identified by any of the other options.

Pyelonephritis is usually caused by which type of organism? a. Bacteria c. Viruses b. Fungi d. Parasites

A Pyelonephritis is usually caused by the bacteria Escherichia coli, Proteus, or Pseudomonas.

Renal cell carcinoma, classified as clear cell tumors, arises from epithelial cells in which structure? a. Proximal tubule c. Nephron b. Distal tubule d. Glomerulus

A Renal cell carcinoma, classified as clear cell tumors according to cell type and extent of metastasis, arises from the proximal tubular epithelium. These tumors are not associated with the other options.

What is the major concern regarding the treatment of gonococci infections? a. Development of antibiotic resistance c. Changes in pathogenicity b. Changes in virulence d. Mutations into different strains

A Several types of drug-resistant strains have been identified; they are penicillinase-producing Neisseria gonorrhoeae (PPNG), which is resistant to penicillin; tetracycline-resistant N. gonorrhoeae (TRNG), which is resistant to tetracycline; chromosomal control of mechanisms of resistance of N. gonorrhoeae (CMRNG), which is resistant to penicillin and tetracycline; and increasingly a fluoroquinolone-resistant N. gonorrhoeae (QRNG). The other options are not major concerns.

What stimulates the desire to eat? a. Agouti-related protein (AgRP) b. Alpha-melanocyte-stimulating hormone (α-MSH) c. Cocaine- and amphetamine-regulated transcript (CART) d. Peptide YY (PYY)

A Specific neurons produce neuropeptide Y (NPY) and AgRP, which stimulates eating and decreases metabolism (anabolic).

Regarding the formation of renal calculi, what function does pyrophosphate, potassium citrate, and magnesium perform? a. They inhibit crystal growth. b. Pyrophosphate, potassium citrate, and magnesium stimulate the supersaturation of salt. c. They facilitate the precipitation of salts from a liquid to a solid state. d. Pyrophosphate, potassium citrate, and magnesium enhance crystallization of salt crystals to form stones.

A Stone or crystal growth inhibiting substances, including potassium citrate, pyrophosphate, and magnesium, are capable of crystal growth inhibition. They are not capable of the functions stated by the other options.

Hypothyroidism, edema, hyperlipidemia, and lipiduria characterize which kidney disorder? a. Nephrotic syndrome c. Chronic glomerulonephritis b. Acute glomerulonephritis d. Pyelonephritis

A Symptoms of nephrotic syndrome include edema, hyperlipidemia, lipiduria, vitamin D deficiency, and hypothyroidism. These symptoms do not support the other options.

During an infection, why do lymph nodes enlarge and become tender? a. B lymphocytes proliferate. b. The nodes are inflamed. c. The nodes fill with purulent exudate. d. The nodes are not properly functioning.

A The B lymphocyte proliferation in response to significant antigen (e.g., during infection) may result in lymph node enlargement and tenderness (reactive lymph node). This description is not accurate for the other options.

increased gastrin secretion by the mother in the last trimester of pregnancy may cause which condition in the infant? a. Pyloric stenosis c. Esophageal atresia b. Meconium ileus d. Galactosemia

A The cause is unknown but increased gastrin secretion by the mother in the last trimester of pregnancy increases the likelihood of pyloric stenosis in the infant. The overproduction of gastric secretions in the infant may be caused by stress-related factors in the mother. This statement is not true of the other options.

The functional kidney is associated with which embryonic organ? a. Metanephros c. Pronephros b. Mesonephros d. Endonephros

A The functional kidney is associated with the metanephros. The kidney develops from three sets of structures: the pronephros (nonfunctional by the end of the embryonic period), mesonephros (nonfunctional), and metanephros (the functional kidney). Endonephros is not relevant to this process.

What does polycythemia at birth indicate? a. Hypoxia in utero c. Congenitally absent spleen b. Dysfunctional bone marrow d. Dehydration in utero

A The hypoxic intrauterine environment stimulates erythropoietin production in the fetus and accelerates fetal erythropoiesis, producing polycythemia (excessive proliferation of erythrocyte precursors) of the newborn. The other options are not related to polycythemia.

The majority of the small percentage of ovarian cancers that are associated with a known pattern of inheritance are associated with: a. Susceptibility of the BRCA1 gene b. Mutations of the BRCA2 gene c. Hereditary nonpolyposis colorectal cancer (HNPCC) syndrome d. Low progesterone levels

A The majority (approximately 90%) of ovarian cancers are sporadic and not associated with a known pattern of inheritance. Of the 5% to 10% that have a familial component, the majority are associated with the breast cancer susceptibility gene 1 (BRCA1) and a smaller number with mutations of the BRCA2 or mismatched repair genes (HNPCC syndrome). Low progesterone levels are not associated with ovarian cancers.

Bronchiolitis tends to occur during the first years of life and is most often caused by what type of infection? a. Respiratory syncytial virus (RSV) c. Adenoviruses b. Influenzavirus d. Rhinovirus

A The most common associated pathogen is RSV, but bronchiolitis may also be associated with adenovirus, rhinovirus, influenza, parainfluenza virus (PIV), and Mycoplasma pneumoniae.

Which of the following is not considered a cause of galactorrhea? a. Proliferation of the lactiferous ducts of the breast b. Hypothyroidism, resulting from a decrease in thyroid-releasing hormone c. Excess prolactin secretion from the pituitary d. Drugs such as high-dose oral contraceptives and phenothiazines

A The most common cause of galactorrhea is nonpuerperal hyperprolactinemia, or excessive amounts of prolactin. A variety of exogenous agents (such as drugs) and disorders can trigger one of these three mechanisms, thereby causing hyperprolactinemia. Hypothyroidism causes increased secretion of hypothalamic thyroid-releasing hormone, which stimulates the release of prolactin from the pituitary. The proliferation of lactiferous breast ducts is not associated with galactorrhea.

Which is a characteristic lesion of secondary syphilis? a. Condylomata lata c. Chancroid b. Gummas d. Donovan bodies

A The only secondary syphilis lesion is the condylomata lata.

How does the epididymis become infected? a. The pathogenic microorganisms ascend the vasa deferentia from an already infected urethra or bladder. b. The pathogenic microorganisms are attached to sperm that travel through the genital tract. c. The pathogenic microorganisms from the tunica vaginalis are transported to the epididymis. d. The pathogenic microorganisms from the prostate fluid ascend to the epididymis.

A The pathogenic microorganisms usually reach the epididymis by ascending the vasa deferentia from an already infected urethra or bladder. Of the selections available, this is the only option that accurately describes how the epididymis becomes infected.

Clinical manifestations of decreased exercise tolerance, wheezing, shortness of breath, and productive cough are indicative of which respiratory disorder? a. Chronic bronchitis c. Pneumonia b. Emphysema d. Asthma

A The symptoms that lead individuals with chronic bronchitis to seek medical care include decreased exercise tolerance, wheezing, and shortness of breath. Individuals usually have a productive cough ("smoker's cough"). The described symptoms are not associated with any of the other options.

Cirrhosis causes intrahepatic portal hypertension in children as a result of which mechanism? a. Fibrosis that increases the resistance to blood flow within the portal system b. Increased pressure from the twisting of the common bile ducts c. Development of collateral circulation within the portal system d. Shunting of fluid to the spleen or abdomen

A The two basic causes of portal hypertension in children are (1) increased resistance to blood flow within the portal system and (2) increased volume of portal blood flow. This selection is the only option that correctly identifies the cause of intrahepatic portal hypertension in children.

What is the cause of smoky, brown-colored urine resulting from acute poststreptococcal glomerulonephritis? a. Presence of red blood cells c. Slough from the collecting tubules b. Presence of urobilinogen d. Protein in the urine

A The urine is usually smoky brown or cola-colored because of the presence of red blood cells. The other options are not correct statements regarding this condition.

Recycling of iron from erythrocytes is made possible by which of the following? a. Transferrin c. Apoferritin b. Hemosiderin d. Erythropoietin

A Transferrin is recycled (transferrin cycle) in the following manner: (1) the transferrin-iron complex binds to a transferring receptor on the erythroblast's plasma membrane; (2) the complex moves into the cell by receptor-mediated endocytosis; (3) iron is released (dissociated) from transferrin; and (4) the dissociated transferrin is returned to the bloodstream for reuse. The other options do not present an accurate description of the recycling of erythrocytic iron.

Which condition is not a cause of chest wall restriction? a. Pneumothorax c. Gross obesity b. Severe kyphoscoliosis d. Neuromuscular disease

A Unlike the other options that result in chest wall restriction, a pneumothorax is the presence of air or gas in the pleural space caused by a rupture in the visceral pleura (which surrounds the lungs) or the parietal pleura and chest wall.

Which urine characteristics are indicative of acute tubular necrosis (ATN) caused by intrinsic (intrarenal) failure? a. Urine sodium >30 mEq/L b. Urine osmolality >500 mOsm c. Fractional excretion of sodium (FENa) <1% d. Urine sediment has no cells, some hyaline casts

A Urine sodium >30 mEq/L is the only option indicative of ATN.

In what form of bronchiectasis do both constrictions and dilations deform the bronchi? a. Varicose c. Cylindric b. Symmetric d. Saccular

A Varicose bronchiectasis exists when both constrictions and dilations deform the bronchi. None of the other options involve both constriction and dilation, resulting in bronchi deformity.

What is the mechanism for developing Wilms tumor? a. The development of a Wilms tumor involves tumor-suppressor genes located on chromosome 11. b. Development involves an autosomal dominant inherited disorder involving the Y chromosome. c. Wilms tumor is an autoimmune disorder. d. The development of a Wilms tumor is a congenital anomaly.

A Wilms tumor-suppressor genes WT1 and WT 2 are located on chromosome 11. The other options are not considered relevant to the development of a Wilms tumor.

Which characteristics are symptomatic of a flail chest? (Select all that apply.) a. Involves the fracture of several consecutive ribs. b. Involves multiple fractures to individual ribs. c. Can involve the fracture of the sternum. d. Is generally a result of the inflammatory process. e. Is more common among the older adult population.

A, B, C A flail chest results from the fracture of several consecutive ribs in more than one place or the fracture of the sternum and several consecutive ribs. Age and inflammation are not generally considered factors in this disorder.

What are the classic symptoms associated with hepatitis A in children? (Select all that apply.) a. Nausea b. Vomiting c. Diarrhea d. Jaundice e. Muscle pain

A, B, C Clinical manifestations associated with hepatitis A may include nausea, vomiting, and diarrhea. Neither jaundice nor muscle pain is associated with hepatitis A in children.

What are considered risk factors for developing bladder and kidney cancers? (Select all that apply.) a. Cigarette smoking b. Hypertension c. Exposure to aniline dyes d. Below normal body weight e. Male gender

A, B, C Risk factors for renal cancer include cigarette smoking, obesity, and hypertension. The risk of primary bladder cancer is greater among people who smoke or those who are exposed to metabolites of aniline dyes or other aromatic amines or chemicals and with heavy consumption of phenacetin. Gender is not a recognized risk factor.

Which statements are true regarding the role of the endothelium in clot formation? (Select all that apply.) a. The surface of the endothelium produces plasma protease inhibitors. b. Plasma protease inhibitors assist in preventing clot formation. c. Thrombomodulin is a protein that is converted on the surface of endothelial cells. d. Protein A binds to thrombomodulin. e. Activated protein C enhances the adhesion ability of neutrophils.

A, B, C The surface of the endothelium produces plasma protease inhibitors to resist clot formation. Thrombomodulin is a membrane thrombin-binding protein matter and is converted to activated protein C (see Figure 27-18) on the surface of endothelial cells. Protein C in the circulation binds to thrombomodulin. Activated protein C inhibits the adhesion of neutrophils to the endothelium.

What is a recognized treatment for the symptoms often associated with pelvic organ prolapse? (Select all that apply.) a. Pessary b. Kegel exercises c. Estrogen therapy d. Surgical repair e. Bearing down exercises

A, B, C, D A common first-line treatment is a pessary, which is a removable mechanical device that holds the uterus in position. The pelvic fascia may be strengthened through Kegel exercises (repetitive isometric tightening and relaxing of the pubococcygeal muscles) or by estrogen therapy in menopausal women. Maintaining a healthy body mass index, preventing constipation, and treating chronic cough may help as well. Surgical repair with or without a hysterectomy is the treatment of last resort. Bearing down would likely exacerbate the problem.

Which inflammatory mediators are produced in asthma? (Select all that apply.) a. Histamine b. Bradykinin c. Leukotrienes d. Prostaglandins e. Neutrophil proteases

A, B, C, D A large number of inflammatory mediators, such as histamine, prostaglandins, and leukotrienes, are produced by asthma. Neutrophil proteases are not produced in relationship to asthma.

Which statements are true regarding the hepatitis B infection? (Select all that apply.) a. Hepatitis B poses a significant risk for chronic liver disease. b. Hepatocellular cancer is a common comorbid condition. c. Universal vaccination of infants and children is recommended. d. The preventive method of choice is immunization. e. Universal vaccination of sexually active adults is recommended.

A, B, C, D Hepatitis B infection poses significant health risks including chronic liver disease and hepatocellular cancer. Immunization against hepatitis B is the most effective means of preventing transmission. Universal vaccination of infants and children is recommended, as well as vaccination of high-risk adults.

Which renal disorders are considered causes of intrarenal renal failure? (Select all that apply.) a. Acute glomerulonephritis b. Allograft rejection c. Tumors d. Acute tubular necrosis (ATN) e. Prostatic hypertrophy

A, B, C, D Intrarenal (intrinsic) acute kidney injury (AKI) may result from ischemic ATN, nephrotoxic ATN, acute glomerulonephritis, vascular disease, allograft rejection, or interstitial disease (drug allergy, infection, tumor growth). Prostatic hypertrophy is not associated with intrarenal renal failure.

What are the common modes of transmission for the hepatitis B virus (HBV)? (Select all that apply.) a. Needle punctures b. Blood transfusions c. Contact with infected body fluids d. Skin cuts e. Ingestion of infected substances

A, B, C, D Transmission of HBV can occur through needle puncture, blood transfusion, cuts in the skin, and contact with infected body fluids. Ingestion is not a recognized transmission mode.

Which cells of the inflammatory process are found in acute poststreptococcal glomerulonephritis? (Select all that apply.) a. Immunoglobulin G (IgG) b. Immunoglobulin A (IgA) c. Complement C3 d. Immunoglobulin E (IgE) e. Immune complexes

A, B, C, E Antigen-antibody complexes of IgG, IgA, and C3 complement are deposited in the glomerulus, or the antigen may be trapped within the glomerulus and immune complexes formed in situ. IgE is not relevant to this situation.

Which statements are true regarding the narrow chemical safety margin that infants demonstrate? (Select all that apply.) a. The immaturity of tubules may diminish the response to antidiuretic hormone (ADH). b. An immature tubular transport capacity impairs the excretion of potassium. c. An immature tubular transport capacity impairs the reabsorption of bicarbonate. d. The immaturity of tubules may diminish the response to aldosterone. e. The safety margin begins to expand after 2 years of age.

A, B, C, E Because of a high hydrogen ion concentration, a limited ability to regulate the internal environment, and a lowered osmotic pressure, the infant's renal system has a narrow chemical safety margin. The immaturity and smaller surface area of the tubules also may diminish the water reabsorption response to ADH. An immature tubular transport capacity means that the ability to excrete a potassium load, reabsorb bicarbonate, or buffer hydrogen with ammonia does not become efficient until approximately 2 years of age.

Which clinical manifestations are consistent with cancer of the cecum and ascending colon? (Select all that apply.) a. Mahogany-colored blood mixed with stool b. Anemia c. Pain d. Constipation e. Palpable mass in the lower right quadrant

A, B, C, E Clinical manifestations consistent with cancer of the cecum and ascending colon include pain, a palpable mass in the lower right quadrant, anemia, and dark red or mahogany-colored blood mixed with the stool. Constipation is not associated with this diagnosis.

Which situations have been associated with possible causes of the failure to thrive (FTT) in infants? (Select all that apply.) a. Gastroesophageal reflux b. Pyloric stenosis c. Intestinal parasites d. Adoption at birth e. Psychosocial isolation

A, B, C, E FTT is a disorder having organic (e.g., gastrointestinal and endocrine disorders) and nonorganic (e.g., psychosocial) deprivation causes. Organic FTT has a pathophysiologic cause, such as gastroesophageal reflux, pyloric stenosis, gastroenteritis, malabsorption syndromes, infection by intestinal parasites, congenital anomalies, very low birth weight, or chronic diseases of major body systems. Nonorganic FTT occurs in the absence of any gastrointestinal, endocrine, or other chronic diseases. It is usually associated with psychosocial deprivation, although behavior problems may contribute to its occurrence in the absence of maternal pathologic findings. Adoption, in of itself, is not a cause.

Which statements about plasma proteins are correct? (Select all that apply.) a. Provide clotting factors. b. Transport triglycerides. c. Synthesize complement proteins. d. Create hydrostatic pressure. e. Transport cholesterol.

A, B, C, E Plasma proteins do not create hydrostatic pressure. The other options are all accurate statements regarding plasma proteins.

Which statements are true regarding urethritis? (Select all that apply.) a. A purulent drainage may be present. b. A clear mucus-like discharge may be present. c. Symptoms include urethral tingling and itching or burning on urination. d. A 24-hour urine test is required to diagnose the disorder. e. Treatment includes appropriate antibiotic therapy.

A, B, C, E Symptoms of urethritis include urethral tingling and itching or a burning sensation on urination (dysuria), frequency, and urgency. The individual may note a purulent or clear mucus-like discharge from the urethra. Nucleic acid detection amplification tests allow easy detection of Neisseria gonorrhoeae and Chlamydia trachomatis in first-void urine. Treatment consists of appropriate antibiotic therapy for infectious urethritis and an avoidance of future chemical or mechanical irritation.

What are the clinical manifestations of bacterial pneumonia in children? (Select all that apply.) a. Fever with chills b. Productive cough c. Dyspnea d. Respiratory alkalosis e. Malaise

A, B, C, E The clinical presentation of bacterial pneumonia, particularly pneumococcal, may include a preceding viral illness, followed by fever with chills and rigors, shortness of breath, and an increasingly productive cough. Auscultation usually reveals such abnormalities as crackles or decreased breath sounds. Other less specific findings may include malaise, emesis, abdominal pain, and chest pain. Respiratory alkalosis is not usually associated with bacterial pneumonia in children.

Which are clinical manifestations of male breast cancer? (Select all that apply.) a. Ulceration present on the breast b. Retraction of breast tissue c. Nipple discharge d. Palpable mass midline of the nipple e. Unilateral solid mass

A, B, C, E The malignant male breast lesion is usually a unilateral solid mass located near the nipple. Because the nipple is commonly involved, crusting and nipple discharge are typical clinical manifestations. Other findings include skin retraction, ulceration of the skin over the tumor, and axillary node involvement.

Which statements are true regarding hemolytic uremic syndrome (HUS)? (Select all that apply.) a. Microangiopathic hemolytic anemia characterizes HUS. b. Thrombocytopenia is a comorbid condition. c. Wilms tumors are often the trigger for HUS. d. HUS is the most common community-acquired cause of acute renal failure in young children. e. HUS is chronic in nature.

A, B, D HUS is an acute disorder characterized by microangiopathic hemolytic anemia and thrombocytopenia and is the most common community-acquired cause of acute renal failure in young children. Wilms tumors are not necessarily associated with this disorder.

Which structural anomalies are associated with urinary tract malformations? (Select all that apply.) a. Low-set ears b. Prune-belly syndrome c. Broad-bridged nose d. Imperforate anus e. Malformed lips

A, B, D Structural anomalies that are associated with urinary tract malformations include low-set, malformed ears; absent abdominal muscles (prune-belly syndrome); and imperforate anus or genital deviation. Anomalies of the either the nose or the lips are not generally associated with urinary tract malformations.

What are the primary anticoagulant mechanisms? (Select all that apply.) a. Antithrombin III b. Tissue factor pathway inhibitor c. Hematopoiesis d. Protein C e. Phagocytosis

A, B, D The major regulatory factors that control hemostasis reside where the greatest probability of clotting would occur—on the endothelial cell surface. The primary anticoagulant mechanisms include thrombin inhibitors (e.g., antithrombin III), tissue factor inhibitors (e.g., tissue factor pathway inhibitor), and mechanisms for degrading activated clotting factors (e.g., protein C). Hematopoiesis and phagocytosis are processes that are not related to anticoagulation.

Children diagnosed with chronic asthma are likely to exhibit which symptoms? (Select all that apply.) a. Nasal flaring b. Musical expiratory wheezing c. Clubbing of fingers and toes d. Substernal retractions e. Diaphoresis

A, B, D, E On physical examination, expiratory wheezing that is often described as high pitched and musical is exhibited, along with prolongation of the expiratory phase of the respiratory cycle. Hyperinflation is sometimes visible. The respiratory rate is elevated, as is the heart rate. Nasal flaring and accessory muscle use are evident, with retractions in the substernal, subcostal, intercostal, suprasternal, or sternocleidomastoid areas. Infants may appear to be "head bobbing" because of sternocleidomastoid muscle use. Pulsus paradoxus may also be present. The child may appear anxious or diaphoretic, which are important signs of respiratory compromise. Clubbing of fingers and toes is not typically associated with asthma.

Which statements are true regarding parietal pain? (Select all that apply.) a. Parietal pain arises from the parietal peritoneum. b. It is generally more localized than visceral pain. c. Parietal pain is usually less intense than visceral pain. d. Nerve fibers that travel to the spinal cord are involved in parietal pain. e. Parietal pain corresponds to dermatomes T6 and L1.

A, B, D, E Parietal pain arises from the parietal peritoneum and is more localized and intense than visceral pain. Nerve fibers from the parietal peritoneum travel with peripheral nerves to the spinal cord, and the sensation of pain corresponds to skin dermatomes T6 and L1.

The size of benign uterine tumors, such as leiomyomas, is thought to be caused by the influence of which hormone? (Select all that apply.) a. Progesterone b. Estrogen c. Luteinizing hormone d. Gonadotropin-stimulating hormone e. Growth factors

A, B, E The cause of uterine leiomyomas is unknown, although their size appears to be related to only estrogen, progesterone, growth factors, angiogenesis, and apoptosis.

Which are the early (prodromal) clinical manifestations of hepatitis? (Select all that apply.) a. Fatigue b. Vomiting c. Itching d. Splenomegaly e. Hyperalgia

A, B, E The prodromal (preicteric) phase of hepatitis begins approximately 2 weeks after exposure and ends with the appearance of jaundice. Fatigue, anorexia, malaise, nausea, vomiting, headache, hyperalgia, cough, and low-grade fever are prodromal symptoms that precede the onset of jaundice. Itching and splenomegaly are not associated with the prodromal phase of hepatitis.

What problems are commonly associated with the diagnosis of horseshoe kidneys? (Select all that apply.) a. Hydronephrosis b. Nephritis c. Infection d. Kidney stones e. Pyuria

A, C, D Approximately one-third of individuals with horseshoe kidneys are asymptomatic, and the most common problems are hydronephrosis, infection, and stone formation. Nephritis and pyuria are not associated with this diagnosis.

What are the causes of dyspnea? (Select all that apply.) a. Decreased pH, increased partial pressure of arterial carbon dioxide (PaCO2) and decreased partial pressure of arterial oxygen (PaO2) b. Decreased blood flow to the medulla oblongata c. Stimulation of stretch or J-receptors d. Presence of anxiety e. Presence of pain

A, C, D Dyspnea can be triggered by decreased pH, increased PaCO2, and decreased PaO2. Stimulation of either stretch or J-receptors is also known as a cause of dyspnea. Dyspnea may be the result of pulmonary disease or many other conditions, such as pain, heart disease, trauma, and anxiety. No data are available to support the role of decreased blood flow to the medulla oblongata as being a cause of dyspnea.

What factors can contribute to the development of a cleft lip and a cleft palate? (Select all that apply.) a. Maternal deficiency of B vitamins b. Exposure to heavy metals during the second trimester of pregnancy c. Maternal use of tobacco d. Maternal diabetes mellitus e. Genetic mutation of the transforming growth factor

A, C, D In most cases, cleft lip and cleft palate are caused by multiple gene and environmental interactions, including maternal deficiency of B vitamins (B6, folic acid, and B12), maternal tobacco and alcohol use, maternal diabetes mellitus, and genetic variations of several biomolecules including transforming growth factor, interferon regulatory factor-6, fibroblast growth factor, and other growth factors. Exposure to heavy metals during the second trimester of pregnancy is not a known trigger for such development.

Which statements regarding Mycobacterium tuberculosis are true regarding the bacilli's ability to go into dormancy? (Select all that apply.) a. Neutrophils and macrophages all play a role in its dormancy. b. Mycobacterium tuberculosis is capable of dormancy but for only a short period. c. The immune system is the controlling factor regarding its length of dormancy. d. The bacilli are sealed off in tubercles to allow for dormancy. e. An attack by lymphocytes brings the bacilli out of their dormant state.

A, C, D Neutrophils, lymphocytes, and macrophages seal off the colonies of bacilli, forming a granulomatous lesion called a tubercle. Once the bacilli are isolated in tubercles and immunity develops, tuberculosis may remain dormant for life. If the immune system is impaired, however, or if live bacilli escape into the bronchi, active disease occurs and may spread through the blood and lymphatic system to other organs. This microorganism can remain dormant for extended periods. Reverting from dormancy is not related to a lymphocyte attack.

Which statements are true regarding the parasitic infection referred to as scabies? (Select all that apply.) a. Scabies is spread through skin-to-skin contact. b. The crab lice, Phthirus pubis, cause scabies. c. Severe pruritus is its major clinical manifestation. d. Symptoms worsen at night. e. Treatment is provided through oral medication therapy.

A, C, D Scabies is a common parasitic infection that can be spread by skin-to-skin contact and sexual contact. The scabies mite burrows through the skin, depositing two or three large eggs per day. Intense pruritus, especially at night, is the most pronounced clinical manifestation. Treatment consists of topical application of a pediculicide. Pediculosis pubis (crabs) is commonly transmitted sexually and is caused by the crab louse, P. pubis.

Which statements are true concerning struvite stones? (Select all that apply.) a. They are more common in women than in men. b. Struvite stones are associated with chronic laxative use in women. c. They grow large and branch into a staghorn configuration in renal pelvis and calyces. d. Struvite stones are closely associated with urinary tract infections caused by urease-producing bacteria, such as Pseudomonas. e. They are more common in men than in women.

A, C, D Women are at greater risk for developing struvite stones, but the risk is greater because women have an increased incidence of urinary tract infections not because of chronic laxative use. Such stones grow large and branch into a staghorn configuration and are associated with Pseudomonas.

Which statements are true regarding chronic gastritis? (Select all that apply.) a. Chronic gastritis tends to occur in older adults. b. It causes thinning and degeneration of the stomach wall. c. Chronic gastritis results in chronic inflammation and mucosal atrophy. d. Mucosal atrophy is a common outcome of chronic gastritis. e. Epithelial metaplasia is often observed with chronic gastritis.

A, C, D, E Chronic gastritis tends to occur in older adults and causes chronic inflammation, mucosal atrophy, and epithelial metaplasia. Neither thinning nor degeneration of the stomach wall is associated with chronic gastritis

Which statements are true regarding exudative effusion? (Select all that apply.) a. Exudative effusion contains high concentrations of white blood cells. b. Exudative effusion produces a very thick exudate. c. Exudative effusion may occur in response to an inflammatory process. d. The presence of a malignant cancer can trigger exudative effusion. e. Exudative effusion is a result of increased capillary permeability.

A, C, D, E Exudative effusion is less watery and contains high concentrations of white blood cells and plasma proteins. Exudative effusion occurs in response to inflammation, infection, or malignancy and involves inflammatory processes that increase capillary permeability.

Which symptoms are considered part of the nephrotic syndrome in children? (Select all that apply.) a. Proteinuria b. Pyuria c. Hyperlipidemia d. Edema e. Hypoalbuminemia

A, C, D, E Nephrotic syndrome is a term used to describe a symptom complex characterized by proteinuria, hypoalbuminemia, hyperlipidemia, and edema. Pyuria is not observed in this condition.

Which factors are involved in the development of hypospadias? (Select all that apply.) a. Disruption in male hormones, including testosterone b. Mutation of gene HP-1 (chromosome 16) c. 5α-reductase mutations d. Hormones administered for in vitro fertilization e. Advanced maternal age

A, C, D, E The cause of hypospadias is multifactorial and related to disruptions in male hormones, including testosterone biosynthesis defects, 5α-reductase mutations, hormones administered for in vitro fertilization, advanced maternal age, and other environmental factors. A mutation of gene HP-1 (chromosome 16) is not relevant.

Which statements regarding Wilson disease in children are true? (Select all that apply.) a. Wilson disease is a rare autosomal recessive defect. b. Wilson disease affects copper metabolism. c. A lack of necessary copper is a result of Wilson disease. d. The liver is often affected in Wilson disease. e. Corneal damage can be a result of Wilson disease.

A, C, D, E Wilson disease (hepatolenticular degeneration) is an autosomal recessive defect of copper metabolism that causes toxic amounts of copper to accumulate in the liver, brain, kidneys, and corneas

Which hormones are natural appetite suppressants? (Select all that apply.) a. Insulin b. Cortisol c. Galanin d. Calcitonin e. Serotonin

A, D, E Insulin, calcitonin, and serotonin are natural appetite suppressants, whereas cortisol and galanin are natural appetite stimulants.

Which statement is false regarding the pathophysiologic process of acute pancreatitis? a. Bile duct or pancreatic duct obstruction blocks the outflow of pancreatic digestive enzymes. b. Acute pancreatitis can also result from direct cellular injury from drugs or viral infection. c. Acute pancreatitis is an autoimmune disease in which immunoglobulin G (IgG) coats the pancreatic acinar cells; consequently, the pancreatic enzymes destroy the cells. d. Acute pancreatitis is usually mild and spontaneously resolves.

C The backup of pancreatic secretions and the activation and release of enzymes (activated trypsin activates chymotrypsin, lipase, and elastase) within the pancreatic acinar cells cause acute pancreatitis, an obstructive disease. The activated enzymes cause autodigestion (e.g., proteolysis, lipolysis) of the pancreatic cells and tissues, resulting in inflammation. Acute pancreatitis is usually a mild disease and spontaneously resolves; however, approximately 20% of those with the disease develop a severe acute pancreatitis that requires hospitalization. Pancreatitis develops because of a blockage to the outflow of pancreatic digestive enzymes caused by bile duct or pancreatic duct obstruction (e.g., gallstones). Acute pancreatitis can also result from direct cellular injury from drugs or viral infection.

Which sexually transmitted infection frequently coexists with gonorrhea? a. Syphilis c. Chlamydia b. Herpes simplex virus d. Chancroid

C The coexistence of chlamydial infection with gonorrhea frequently occurs. No coexistence exists with the other options.

A patient exhibits symptoms including hematuria with red blood cell casts and proteinuria exceeding 3 to 5 g/day, with albumin as the major protein. These data suggest the presence of which disorder? a. Cystitis c. Glomerulonephritis b. Chronic pyelonephritis d. Nephrotic syndrome

C The data suggest the patient has the disorder known as glomerulonephritis. Two major changes distinctive of more severe glomerulonephritis are (1) hematuria with red blood cell casts and (2) proteinuria exceeding 3 to 5 g/day with albumin as the major protein. These symptoms do not support the diagnosis of the other options.

With which medical diagnosis is meconium ileus often associated? a. Muscular dystrophy c. Cystic fibrosis b. Cerebral palsy d. Congenital aganglionic megacolon

C The detection of albumin in meconium has been used as a screening test for cystic fibrosis. This condition is not associated with any of the other options.

What is the first sign of puberty in boys? a. Thickening of the scrotal skin c. Enlargement of the testes b. Growth of pubic hair d. Change in voice

C The first sign of puberty in boys is an enlargement of the testes and a thinning of the scrotal skin

What initiates inflammation in acute poststreptococcal glomerulonephritis? a. Lysosomal enzymes b. Endotoxins from Streptococcus c. Immune complexes d. Immunoglobulin E (IgE)-mediated response

C The immune complexes initiate inflammation and glomerular injury in acute poststreptococcal glomerulonephritis. Antigen-antibody complexes are deposited in the glomerulus, or the antigen may be trapped within the glomerulus and immune complexes formed in situ. The other options are not involved in initiating inflammation in this situation.

Which type of diarrhea results from lactose intolerance? a. Secretory c. Osmotic b. Motility d. Small volume

C The malabsorption of lactose results in osmotic diarrhea, in which fluids move by osmosis from the vascular compartment into the intestinal lumen. Of the available options, this selection is the only type of diarrhea that results from lactose intolerance.

High altitudes may produce hypoxemia through which mechanism? a. Shunting c. Decreased inspired oxygen b. Hypoventilation d. Diffusion abnormalities

C The presence of adequate oxygen content of the inspired air is the first factor. Oxygen content is lessened at high altitudes. At high altitudes none of the remaining options would be the cause of hypoxemia.

What is the primary problem resulting from respiratory distress syndrome (RDS) of the newborn? a. Consolidation c. Atelectasis b. Pulmonary edema d. Bronchiolar plugging

C The primary problem is atelectasis, which causes significant hypoxemia and is difficult for the neonate to overcome because a significant negative inspiratory pressure is required to open the alveoli with each breath. None of the other options are considered a primary problem associated with RDS.

Which description is used when a progressive neoplastic change involves the full epithelial thickness of the cervix? a. Cervical intraepithelial neoplasia c. Cervical carcinoma in situ b. Cervical dysplasia d. Invasive carcinoma of the cervix

C The progressive neoplastic changes of cervical cells are classified on a continuum from cervical intraepithelial neoplasia (dysplasia) to cervical carcinoma in situ (full epithelial thickness of the cervix is involved), which is generally a precursor of invasive carcinoma of the cervix to invasive carcinoma of the cervical tissue.

The most common clinical manifestation of portal hypertension is what type of bleeding? a. Rectal c. Esophageal b. Duodenal d. Intestinal

C The vomiting of blood from bleeding esophageal varices is the most common clinical manifestation of portal hypertension.

What is the primary site for uncomplicated local gonococci infections in men? a. Epididymis c. Urethra b. Lymph nodes d. Prostate

C Uncomplicated local infections are observed primarily as urethral infections in men.

Bacteria gain access to the female urinary tract by which means? a. Systemic blood that is filtered through the kidney b. Bacteria traveling from the lymph adjacent to the bladder and kidneys c. Bacteria ascending the urethra into the bladder d. Colonization of the bladder when urine is static

C Urinary tract infections (UTIs) in girls occur as a result of perineal bacteria, especially Escherichia coli, ascending the urethra. None of the other options represent the means by which bacteria gain access to the female urinary tract.

Which hemoglobin is made from oxidized ferric iron (Fe3+) and lacks the ability to bind oxygen? a. Deoxyhemoglobin c. Methemoglobin b. Oxyhemoglobin d. Glycosylated hemoglobin

C Without reactivation by methemoglobin reductase, the Fe3+-containing hemoglobin (methemoglobin) cannot bind oxygen. This capability is not true of the other types of hemoglobin mentioned.

Which clinical manifestation is associated with pulmonary hypertension? (Select all that apply.) a. Systemic blood pressure greater than 130/90 mm Hg b. Rhonchi bilaterally c. Dyspnea on exertion d. Peripheral edema e. Jugular venous distention

C, D, E Symptoms of fatigue, chest discomfort, tachypnea, and dyspnea on exertion, palpitations, and cough are common. Examination may reveal peripheral edema, jugular venous distention, a precordial heave, and accentuation of the pulmonary compartment of the second heart sound. Neither rhonchi nor a systemic blood pressure of 130/90 mm Hg are associated with pulmonary hypertension.

Which characteristics allow erythrocytes to function as gas carriers? (Select all that apply.) a. Permanent shape b. Compactness c. Reversible deformability d. Presence of hyperactive mitochondria e. Biconcavity

C, E A red blood cell (RBC) is a small disk with two unique properties: (1) a biconcave shape and (2) the capacity to be reversibly deformed. The other options are not relevant to the function of gas transport.

Which conditions related to the bladder would result from the effects of lesions of the sacral segments below S1? (Select all that apply.) a. Frequency b. Urge incontinence c. Bladder distension d. Urgency e. Urinary retention

C, E Lesions that involve the sacral micturition center (below S1; may also be termed cauda equina syndrome) or peripheral nerve lesions result in detrusor areflexia (acontractile detrusor), a lower motor neuron disorder. The result is an acontractile detrusor or atonic bladder with retention of urine and distention. The other options are associated with neurologic lesions that occur between C2 and S1

Which foods should be eliminated from the diet for children who have gluten-sensitive enteropathy (celiac sprue)? (Select all that apply.) a. Citrus fruits b. Starchy vegetables c. Cereal grains d. Red meat e. Dairy

C, E Treatment for celiac sprue consists of the immediate and permanent institution of a diet free of cereal grains (e.g., wheat, rye, barley, oats, malt). Lactose intolerance is presumed because of damage to the villi; therefore lactose (milk sugar) is excluded from the diet. The remaining options are not contraindicated on a celiac sprue-related diet.

The excretion of urea is low in infants because of which feature? (Select all that apply.) a. Medullary nephrons with comparatively short loops at this stage of development b. Immature tubular transport capacity, impairing the excretion of urea c. High anabolic state d. Dilute urine as a result of the immaturity of an infant's glomeruli e. Available protein used for physical growth

C, E Urea excretion is low in infants primarily because infants are in a high anabolic state and use their protein for growth. The other options are not involved in their low urea excretion.

How is gonorrhea transmitted from a pregnant woman to her fetus? a. Unbound in the blood via the placenta b. Attached to immunoglobulin G (IgG) via the placenta c. Across amniotic membranes by the direct inoculation with the fetal scalp electrodes during labor monitoring d. Predominately through infected cervical and secretions during the birth process

D A pregnant woman can transmit gonorrhea to her fetus during the birth process. The infection passes from mother to child predominately through infected cervical and vaginal secretions. The transmission is not associated with the other options.

When considering the signs and symptoms of acute respiratory distress syndrome (ARDS), the absence of which condition is considered characteristic? a. Progressive respiratory distress c. Decreased pulmonary compliance b. Bilateral infiltrates d. Heart failure

D ARDS is characterized by progressive respiratory distress, severe hypoxemia refractory to treatment with supplemental oxygen, decreased pulmonary compliance, bilateral infiltrates on chest x-ray imaging, and no evidence of heart failure.

By which structure are mature erythrocytes removed from the bloodstream? a. Liver c. Thymus b. Lymph nodes d. Spleen

D After approximately 100 to 120 days in the circulation, old erythrocytes are removed by tissue macrophages, primarily in the spleen.

Hepatic fat accumulation is observed in which form of cirrhosis? a. Biliary c. Postnecrotic b. Metabolic d. Alcoholic

D Alcoholic cirrhosis is a complex process that begins with fatty infiltration (hepatic steatosis). Fat deposition (deposition of triglycerides) within the liver hepatocytes is primarily caused by increased lipogenesis and decreased fatty acid oxidation by hepatocytes. This selection is the only option that accurately identifies the correct form of cirrhosis.

Which term is used to identify a circumscribed area of suppuration and destruction of lung parenchyma? a. Consolidation c. Empyema b. Cavitation d. Abscess

D An abscess is a circumscribed area of suppuration and destruction of lung parenchyma. The described pathologic abnormality is not associated with the other options.

Compared with an adult, an infant has a greater content of extracellular fluid, as well as a greater rate of fluid exchange. What effect does this have on the fluid balance of a child compared with that of an adult? a. Edema development is less of a problem. b. Overhydration is not difficult to manage. c. Daily fluid requirements are greater. d. The control of dehydration is more difficult.

D An infant not only has a greater content of extracellular fluid, but infants also have a greater rate of fluid exchange. The adult takes in and excretes approximately 2000 ml of water daily, representing 5% of the total body fluid and 14% of the extracellular fluid. In contrast, the infant's daily exchange of 600 to 700 ml of water represents 290% of the total body fluid or nearly 50% of the extracellular volume, making control of dehydration and overhydration more difficult. This fact makes the other options incorrect.

Anemia of chronic renal failure can be successfully treated with which element? a. Intrinsic factor c. Vitamin D b. Vitamin B12 d. Erythropoietin

D Anemia of chronic renal failure can be successfully treated with erythropoietin. Reduced erythropoietin secretion and reduced red cell production are evident in anemia resulting from chronic renal failure. The other options are not relevant to this condition.

Clinical manifestations of inspiratory and expiratory wheezing, dyspnea, nonproductive cough, and tachypnea are indicative of which condition? a. Chronic bronchitis c. Pneumonia b. Emphysema d. Asthma

D At the beginning of an attack, the individual experiences chest constriction, expiratory wheezing, dyspnea, nonproductive coughing, prolonged expiration, tachycardia, and tachypnea. Severe attacks involve the use of accessory muscles of respiration, and wheezing is heard during both inspiration and expiration. The presentations of none of the other options are consistent with the described symptoms.

Symptoms of benign prostatic hyperplasia (BPH) are a result of which pathophysiologic condition? a. Infection of the prostate c. Ischemia of the urethra b. Obstruction of the urethra d. Compression of the urethra

D BPH becomes problematic as prostatic tissue compresses the urethra, where it passes through the prostate. Of the selections available, only this option accurately describes the pathophysiologic condition behind the symptoms of BPH.

Which infection has clinical manifestations that include the sudden onset of malaise, low back pain, and perineal pain with high fever and chills, dysuria, nocturia, and urinary retention? a. Orchitis c. Epididymitis b. Balanitis d. Bacterial prostatitis

D Bacterial prostatitis can exhibit common manifestations that include a sudden onset of malaise, low back and perineal pain, high fever (up to 40° C [104° F]), and chills, as well as dysuria, inability to empty the bladder, nocturia, and urinary retention. Myalgia and arthralgia also may occur. This selection is the only option that exhibits the symptoms described.

Granulocytes that contain granules of vasoactive amines, such as histamine, are called: a. Neutrophils c. Monocytes b. Eosinophils d. Basophils

D Basophils contain cytoplasmic granules that hold an abundant mixture of biochemical mediators, including histamine, chemotactic factors, proteolytic enzymes, and an anticoagulant (heparin) (see Figure 27-3, C). This is not an accurate description of any of the other options.

By which method is the organism that causes syphilis best identified? a. Acid-fast stain c. In vitro culture b. Gram-stained slide d. Darkfield microscopy

D Because Treponema pallidum cannot be cultured in vitro, early definitive diagnosis of primary or secondary syphilis depends on darkfield microscopy of a specimen taken from a chancre, regional lymph node, or other lesion. The remaining options are not relevant.

Which statement is false about the causes of enuresis? a. A maturational lag may cause enuresis. b. Enuresis may be related to increased light sleep. c. Obstructive sleep apnea may be a symptom of enuresis. d. Excessive nocturnal levels of vasopressin may cause enuresis.

D Children who do not have the normal nocturnal elevation of vasopressin produce a higher volume of urine with a lower osmolality. The other options are accurate statements regarding enuresis

Which differentiating sign is required to make the diagnosis of pyelonephritis from that of cystitis? a. Difficulty starting the stream of urine b. Spasmodic pain that radiates to the groin c. Increased glomerular filtration rate d. Urinalysis confirmation of white blood cell casts

D Clinical assessment, alone, is difficult to differentiate the symptoms of cystitis from those of pyelonephritis. Urine culture, urinalysis, and clinical signs and symptoms establish the specific diagnosis. White blood cell casts indicate pyelonephritis, but they are not always present in the urine. This selection is the only option that is considered a required sign of pyelonephritis.

What medical term is used for a condition that results from pulmonary hypertension, creating chronic pressure overload in the right ventricle? a. Hypoxemia c. Bronchiectasis b. Hypoxia d. Cor pulmonale

D Cor pulmonale develops as pulmonary hypertension and creates chronic pressure overload in the right ventricle similar to that created in the left ventricle by systemic hypertension. None of the other options identify the condition.

Cryptorchidism can be defined as which of the following? a. Normal developmental state of the testes b. Abnormal state in which the testes are overdeveloped c. Lack of scrotum d. Testicular maldescent

D Cryptorchidism is a condition of testicular maldescent, the only option that accurately defines cryptorchidism.

Which condition is considered a clinical cause of amenorrhea? a. Disorder in the endometrium c. Lack of physical exercise b. Obstruction of the fallopian tubes d. Failure to ovulate

D Depressed ovarian hormone levels, which are associated with a variety of clinical disorders, also cause amenorrhea by preventing ovulation. This option is the only answer that accurately identifies a clinical cause of cycle irregularities.

Pulmonary edema and pulmonary fibrosis cause hypoxemia by which mechanism? a. Creating alveolar dead space b. Decreasing the oxygen in inspired gas c. Creating a right-to-left shunt d. Impairing alveolocapillary membrane diffusion

D Diffusion of oxygen through the alveolocapillary membrane is impaired if the alveolocapillary membrane is thickened or if the surface area available for diffusion is decreased. Abnormal thickness, as occurs with edema (tissue swelling) and fibrosis (formation of fibrous lesions), increases the time required for diffusion across the alveolocapillary membrane. None of the remaining options accurately describes the mechanism that triggers hypoxemia as a result of pulmonary edema or pulmonary fibrosis.

What is the most immediate result of a small intestinal obstruction? a. Vomiting c. Electrolyte imbalances b. Dehydration d. Distention

D Distention begins almost immediately, as gases and fluids accumulate proximal to the obstruction. Within 24 hours, up to 8 L of fluid and electrolytes enters the lumen in the form of saliva, gastric juice, bile, pancreatic juice, and intestinal secretions. Copious vomiting or sequestration of fluids in the intestinal lumen prevents their reabsorption and produces severe fluid and electrolyte disturbances.

After a partial gastrectomy or pyloroplasty, clinical manifestations that include increased pulse, hypotension, weakness, pallor, sweating, and dizziness are the results of which mechanism? a. Anaphylactic reaction in which chemical mediators, such as histamine, prostaglandins, and leukotrienes, relax vascular smooth muscles, causing shock b. Postoperative hemorrhage during which a large volume of blood is lost, causing hypotension with compensatory tachycardia c. Concentrated bolus that moves from the stomach into the small intestine, causing hyperglycemia and resulting in polyuria and eventually hypovolemic shock d. Rapid gastric emptying and the creation of a high osmotic gradient in the small intestine, causing a sudden shift of fluid from the blood vessels to the intestinal lumen

D Dumping syndrome occurs with varying severity in 5% to 10% of individuals who have undergone partial gastrectomy or pyloroplasty. Rapid gastric emptying and the creation of a high osmotic gradient in the small intestine cause a sudden shift of fluid from the vascular compartment to the intestinal lumen. Plasma volume decreases, causing vasomotor responses, such as increased pulse rate, hypotension, weakness, pallor, sweating, and dizziness. Rapid distention of the intestine produces a feeling of epigastric fullness, cramping, nausea, vomiting, and diarrhea. This selection is the only option that accurately identifies the mechanism responsible for the described situation.

Which of the following is the role of nitric oxide (NO) in hemostasis? a. Stimulates the release of fibrinogen to maintain the platelet plug. b. Stimulates the release of clotting factors V and VII. c. Causes vasoconstriction and stimulates platelet aggregation. d. Controls platelet activation through cyclic adenosine monophosphate (cAMP)-mediated signaling.

D Endothelial cell NO synthase produces NO, which controls platelet activation through cAMP-mediated signaling. The other options do not present an accurate description of the role of NO in hemostasis.

In the 95% of those with delayed puberty, the problem is caused by which condition? a. Disruption in the hypothalamus c. Deficit in estrogen or testosterone b. Disruption of the pituitary d. Physiologic hormonal delays

D In 95% of cases, delayed puberty is a physiologic delay; that is, hormonal levels are normal and the hypothalamic-pituitary-gonadal (HPG) axis is intact, but maturation is happening slowly. This selection is the only option that accurately describes 95% of those with delayed puberty.

In 95% of children of delayed puberty, the problem is caused by: a. Disruption in the hypothalamus c. Deficit in estrogen or testosterone b. Disruption of the pituitary d. Physiologic hormonal delays

D In 95% of children with delayed puberty, the delay is physiologic; that is, hormonal levels are normal and the hypothalamic-pituitary-gonadal (HPG) axis is intact, but maturation is slowly happening. This option is the only answer that accurately describes the most common cause of delayed puberty.

In tuberculosis, the body walls off the bacilli in a tubercle by stimulating which action? a. Macrophages that release tumor necrosis factor-alpha (TNF-α) b. Phagocytosis by neutrophils and eosinophils c. Formation of immunoglobulin G to initiate the complement cascade d. Apoptotic infected macrophages that activate cytotoxic T cells

D In defense, macrophages and lymphocytes release interferon, which inhibits the replication of the microorganism and stimulates more macrophages to attack the bacterium. Apoptotic infected macrophages can also activate cytotoxic T cells (cluster of differentiation [CD] 8). Tuberculosis does not trigger the mechanisms described by the other options.

What is the effect of low plasma albumin? a. Clotting factors decrease, thus increasing the chance of prolonged bleeding. b. Fewer immunoglobulins are synthesized, thus impairing the immune function. c. Less iron is stored, thus increasing the incidence of iron deficiency anemia. d. Osmotic pressure decreases, thus water moves from the capillaries to the interstitium.

D In the case of decreased production (e.g., cirrhosis, other diffuse liver diseases, protein malnutrition) or excessive loss of albumin (e.g., certain kidney diseases, extensive burns), the reduced oncotic pressure leads to excessive movement of fluid and solutes into the tissues and decreased blood volume. The other options are not accurate descriptions of the effect of low plasma albumin.

What is usually the first clinical manifestation of breast cancer? a. Nipple dimpling c. Enlargement of one breast b. Nipple discharge d. Painless lump

D Invasive carcinoma of the breast generally exhibits a nontender palpable mass or thickened area. This option is the only answer that accurately describes the first clinical manifestation of breast cancer.

Kussmaul respirations as a respiratory pattern may be associated with which characteristic(s)? a. Alternating periods of deep and shallow breathing b. Pulmonary fibrosis c. Chronic obstructive pulmonary disease d. Slightly increased ventilatory rate, large tidal volumes, and no expiratory pause

D Kussmaul respirations are characterized by a slightly increased ventilatory rate, very large tidal volume, and no expiratory pause. Kussmaul respirations are not associated with any of the other options.

Which statement provides the most accurate information regarding the transmission of herpes simplex virus (HSV)? a. HSV is transmitted only when vesicles are present. b. HSV is transmitted only while lesions are present. c. The use of condoms prevents the transmission of HSV. d. The risk of transmission is present even during latent periods

D Latent infections can become reactivated and cause a recurrent infection with similar manifestations. The other options are not accurate statements regarding the transmission of HSV.

Antiemetic agents, such as domperidone and haloperidol, are antagonists for which receptors? a. 5-Hydroxytryptamine (5-HT) serotonin b. Histamine-2 c. Acetylcholine d. Dopamine

D Metoclopramide, domperidone, and haloperidol are dopamine antagonists, making them effective antiemetic agents. This selection is the only option that identifies a receptor that is involved in the process of vomiting.

Which sexually transmitted disease occasionally causes clinical manifestations of scant intermittent penile discharge, slight pruritus, and mild dysuria? a. Campylobacter enteritis c. Cytomegalovirus enteritis b. Shigellosis d. Trichomoniasis

D Most men with trichomoniasis are asymptomatic but may have scant intermittent discharge, slight pruritus, and mild dysuria. The other options do not demonstration these symptoms.

Without prior exposure to an antigen, which cells are able to destroy some types of tumor cells and some virus-infected cells? a. Lymphocytes c. Megakaryocytes b. Plasma cells d. Natural killer (NK) cells

D NK cells, which resemble large granular lymphocytes, kill some types of tumor cells (in vitro) and some virus-infected cells without being induced by previous exposure to these antigens. This capability is not true of the other options.

What is the cause of functional dysphagia? a. Intrinsic mechanical obstruction c. Tumor b. Extrinsic mechanical obstruction d. Neural or muscular disorders

D Neural or muscular disorders that interfere with voluntary swallowing or peristalsis cause functional dysphagia. This selection is the only option that accurately identifies a cause of functional dysphagia.

Obesity is defined as a body mass index (BMI) greater than what measurement? a. 22 c. 28 b. 25 d. 30

D Obesity is an energy imbalance, with caloric intake exceeding energy expenditure, and is defined as a BMI greater than 30

What type of vomiting is caused by the direct stimulation of the vomiting center by neurologic lesions involving the brainstem? a. Retch c. Duodenal b. Periodic d. Projectile

D Of the available options, only projectile vomiting is caused by the direct stimulation of the vomiting center by neurologic lesions, such as increased intracranial pressure, tumors, or aneurysms involving the brainstem.

Physiologic jaundice in a newborn is caused by: a. Reabsorption of bilirubin in the small intestine b. Impaired hepatic uptake and excretion of bilirubin c. Increased bilirubin production d. Mild conjugated (indirect-reacting) hyperbilirubinemia

D Of the available options, physiologic jaundice in a newborn is caused by mild unconjugated (indirect-reacting) hyperbilirubinemia.

What type of cyst develops when an ovarian follicle is stimulated but no dominant follicle develops and completes the maturity process? a. Follicular c. Corpus albicans b. Corpus luteal d. Benign ovarian

D Only benign cysts of the ovary are produced when a follicle or a number of follicles are stimulated but no dominant follicle develops and completes the maturity process.

Which statement regarding pelvic inflammatory disease (PID) is true? a. An episode of mild PID can decrease the possibility of a successful pregnancy by 80%. b. Such an inflammation results in temporary changes to the ciliated epithelium of the fallopian tubes. c. PID has not been associated with an increased risk of an ectopic pregnancy. d. Contracting this infection increases the risk of uterine cancer.

D PID infection results in permanent changes to the ciliated epithelium of the fallopian or uterine tubes. A recent study has found that one episode of mild, subclinical PID resulted in a 40% decrease in later pregnancy rates, and multiple episodes of PID further increase the risk of infertility. Scarring caused by PID greatly increases the risk of a later ectopic pregnancy by up to tenfold. Scarring and adhesions also can result in chronic pelvic pain and, potentially, an increased risk of later uterine cancer.

What term is used to identify a fibrotic condition that causes lateral curvature of the penis during erection, which is associated with a local vasculitis-like inflammatory reaction and decreased tissue oxygenation? a. Phimosis c. Lateral paraphimosis b. Lateral phimosis d. Peyronie disease

D Peyronie disease (bent nail syndrome) is a fibrotic condition of the tunica albuginea of the penis, resulting in varying degrees of curvature and sexual dysfunction (see Figure 25-2). Although the exact cause is unknown, a local vasculitis-like inflammatory reaction occurs and decreased tissue oxygenation results in fibrosis and calcification. Peyronie disease is the only term used to identify the pathophysiologic condition described.

What term is used to identify a condition in which the foreskin cannot be retracted over the glans penis? a. Paraphimosis c. Prephimosis b. Priapism d. Phimosis

D Phimosis is the only term used to identify the condition in which the foreskin cannot be retracted back over the glans.

What is plasmin's role in the clotting process? a. Stimulates platelet aggregation. b. Inhibits platelet adhesion and aggregation. c. Prevents the conversion of prothrombin to degrade the fibrin within blood clots. d. Degrades the fibrin within blood clots.

D Plasmin (also called fibrinase or fibrinolysin) is a serine protease that degrades fibrin polymers in clots. It is not capable of the functions described in the remaining options

What is the leading cause of infertility in women? a. Pelvic inflammatory disease c. Salpingitis b. Endometriosis d. Polycystic ovary syndrome

D Polycystic ovary syndrome remains one of the most common endocrine disturbances affecting women, especially young women, and is a leading cause of infertility in the United States

Which cytokines activated in childhood asthma produce an allergic response? a. Interleukin (IL)-1, IL-2, and interferon-alpha (IFN-α) b. IL-8, IL-12, and tumor necrosis factor-alpha (TNF-α) c. IL-4, IL-10, and colony-stimulating factor (CSF) d. IL-4, IL-5, and IL-13

D Related to asthma, IL-4 and IL-13 are particularly important for B-cell switching to favor immunoglobulin E (IgE) production, and IL-5 is crucial for local differentiation and enhanced survival of eosinophils within the airways. This selection is the only option that accurately describes how cytokines produce a childhood asthmatic response.

In which stage of syphilis would the following clinical manifestations be found: destructive skin, bone and soft tissue lesions, aneurysms, heart failure, and neurosyphilis? a. Primary c. Latent b. Secondary d. Tertiary

D Stage IV, tertiary syphilis, is the only stage during which significant morbidity and mortality occur, including destructive skin, bone, and soft-tissue lesions

The risk for respiratory distress syndrome (RDS) decreases for premature infants when they are born between how many weeks of gestation? a. 16 and 20 c. 24 and 30 b. 20 and 24 d. 30 and 36

D Surfactant is secreted into fetal airways between 30 and 36 weeks. The other options are not true regarding the timeframe when the risk for RDS decreases.

Which statement is false concerning the accumulation of fluid in the peritoneal cavity? a. Impaired excretion of sodium by the kidneys promotes water retention. b. Decreased oncotic pressure and increased hepatic sinusoidal hydrostatic pressure cause the movement of fluid into the peritoneal cavity. c. Decreased blood flow to the kidneys activates aldosterone, which retains sodium. d. Circulating nitric oxide causes vasoconstriction, which forces fluid from the capillaries into the peritoneal cavity.

D The arterial vasodilation theory proposes that circulating nitric oxide or the release of endotoxin from translocation of intestinal bacteria triggers arterial vasodilation of the splanchnic organs early in the course of cirrhosis and stimulates renal sodium retention through the renin-angiotensin-aldosterone system, increased sympathetic tone, and changes in the intrarenal blood flow. The other options provide accurate information regarding the accumulation of fluid in the peritoneal cavity.

Which blood cells are biconcave in shape and have the capacity to be reversibly deformed? a. Neutrophils c. Eosinophils b. Monocytes d. Erythrocytes

D The erythrocyte's size and shape are ideally suited to its function as a gas carrier. A red blood cell (RBC) is a small disk with two unique properties: (1) a biconcave shape and (2) the capacity to be reversibly deformed. These are characteristics not observed in any of the other options.

Where are alveolar macrophages found? a. Skin c. Gastrointestinal tract b. Breasts d. Lungs

D The lung is the only location for alveolar macrophages.

What is the most common cause of uncomplicated urinary tract infections? a. Staphylococcus c. Proteus b. Klebsiella d. Escherichia coli

D The most common infecting microorganisms are uropathic strains of E. coli (80% to 85%).

What distinguishes kwashiorkor from marasmus? a. All nutrients, proteins, fats, and carbohydrates are reduced in kwashiorkor. b. Physical growth of children is stunted in kwashiorkor but not in marasmus. c. Muscle wasting, diarrhea, low hemoglobin, and infection characterize kwashiorkor. d. Subcutaneous fat, hepatomegaly, and fatty liver are present in kwashiorkor.

D The presence of subcutaneous fat, hepatomegaly, and fatty liver distinguishes kwashiorkor from marasmus. These manifestations are missing in marasmus because caloric intake is not sufficient to support fat synthesis and storage. None of the other options accurately describes the differences among these conditions.

Which structure(s) in acute respiratory distress syndrome (ARDS) release inflammatory mediators such as proteolytic enzymes, oxygen-free radicals, prostaglandins, leukotrienes, and platelet-activating factor? a. Complement cascade c. Macrophages b. Mast cells d. Neutrophils

D The role of neutrophils is central to the development of ARDS. Activated neutrophils release a battery of inflammatory mediators, among them proteolytic enzymes, oxygen-free radicals (superoxide radicals, hydrogen peroxide, hydroxyl radicals), arachidonic acid metabolites (prostaglandins, thromboxanes, leukotrienes), and platelet-activating factor. These mediators cause extensive damage to the alveolocapillary membrane and greatly increase capillary membrane permeability. The described responses are not associated with the other options.

In glomerulonephritis, what damages the epithelial cells resulting in proteinuria? (Select all that apply.) a. Ischemia b. Lysosomal enzymes c. Compression from edema d. Activated complement e. Altered membrane permeability

D, E Activated complement, inflammatory cytokines, oxidants, proteases, and growth factors attack epithelial cells, alter membrane permeability, and cause proteinuria. None of the other options are responsible for this process.

Cystic fibrosis is directly responsible for complications to which structures? (Select all that apply.) a. Muscles b. Kidneys c. Lymph nodes d. Cervix e. Liver

D, E Of the options available, only cervical inflammation and portal hypertension (liver) are complications directly related to cystic fibrosis.

Which symptom is not a clinical manifestation of croup? a. Rhinorrhea b. Sore throat c. Low-grade fever d. Barking cough e. Coarse rhonchi

E Typically, a prodrome of rhinorrhea, sore throat, and low-grade fever is exhibited for a few days with croup. The child then develops the characteristic harsh (seal-like) barking cough, hoarse voice, and inspiratory stridor. Rhonchi are associated with lower respiratory diseases.

In an infant who is 5 weeks old, an increase in bilirubin production and persistent jaundice support which diagnosis? a. Pathologic hyperbilirubinemia c. Hepatitis A b. Physiologic jaundice d. Infantile cirrhosis

Physiologic jaundice develops during the second or third day after birth and usually subsides in 1 to 2 weeks in full-term infants and in 2 to 4 weeks in premature infants. After this development, increased bilirubin values and persistent jaundice indicate pathologic hyperbilirubinemia. This selection is the only option that accurately identifies the diagnosis associated with these symptoms and timeline.

Which treatment is used for trichomoniasis? a. Topical application of 5-Fluorouracil (5-FU) b. Topical application of acyclovir c. Systemic metronidazole d. Systemic tetracycline

c


Related study sets

Chapter 11 - The Nervous System: Integration and Control

View Set

Medical Pharmacology Part Four AES

View Set

Periodontology 4DD SEM1 - MCQs All

View Set

( CH.2)Fundamentals of nursing terms and NCLEX questions.

View Set

LS 23L Pre/PostLab Quiz Questions Compilation

View Set

Lowdermilk, Chapter 23 Physical Adaptations of the Healthy Newborn

View Set

Astronomy Ch 5 - Telescope Fundamentals

View Set